Peds Final Fall 2023

Réussis tes devoirs et examens dès maintenant avec Quizwiz!

What does a Glasgow Coma Scale (GCS) score of 12 in a child with head trauma suggest?

Awareness of surroundings.

A 1-month-old infant is admitted to the hospital for failure to thrive (FTT) secondary to a cardiac condition. Based on the nurse's knowledge of the different types of FTT, this type of FTT is categorized as A) nonorganic. B) organic. C) idiopathic. D) generalized.

B

A 6-month-old infant attempts to pick up a toy using his entire hand. This action would be documented as using a A) pincer grasp. B) palmer grasp. C) prehension. D) gross motor development.

B

A child relates that every time he eats a certain food, he gets a stomachache. No other discernable physical symptoms have been correlated with the food intake. Based on this information, the nurse would suspect that the child may be exhibiting? A) Food refusal behavior B) Food intolerance C) Food allergy D) Food preference

B

A mother is bringing her 4-month-old infant into the clinic for a routine well-baby check. The mother is exclusively breastfeeding. There are no other liquids given to the infant. What vitamin does the nurse anticipate the provider will prescribe for this infant? A) Vitamin B B) Vitamin D C) Vitamin C D) Vitamin K

B

A parent of a 10-month-old infant tells the nurse that the baby cries and screams whenever the infant is left with the grandparents. Based on the nurse's knowledge of growth and development, the nurse's response is A) the infant is most likely spoiled. B) stranger anxiety is common for an infant of this age. C) separation anxiety should have disappeared between 4 and 8 months of age. D) the grandparents are not responsive to infant.

B

A parent states that she takes her child to a person's home for care while she is at work. There are no other children being care for in the person's home. Based on this finding, the nurse would document child care as being A) in-home care. B) family daycare. C) center-based care. D) licensed daycare.

B

If an infant understands that an object, even if out of sight still exists then it would be correct to note that the infant has achieved A) moral comprehension. B) object permanence. C) mental representation. D) demonstrates reflex activity.

B

In terms of play behavior, which observation would be considered to be an abnormal finding based on normal growth and development patterns? A) Playing peek a boo at age at 1 year of age. B) At 1 month of age, they extend arms to be picked up by their parent or caregiver. C) Show preference for a toy by 4 months of age. D) At 8 months of age, child refuses to play with a stranger.

B

Macrominerals refer to those minerals with daily intake requirements greater than 100 mg. Which is a macromineral? A) Iron B) Calcium C) Fluoride D) Selenium

B

Systematic clinical findings associated with bedbugs manifest as A) wheal. B) anaphylaxis. C) rash. D) folliculitis.

B

The nurse in considering ethical dilemmas that may affect delivery of care with regard to pediatric patients, must consider that the patient's well-being is of paramount importance. This concept is best described by A) fairness. B) beneficence. C) applying equity. D) prevention of harm.

B

The nurse is providing education to parents of an infant diagnosed with colic. What would the nurse include in the discharge teaching? A) The child will have to be watched for gastrointestinal issues in the future. B) The symptoms of colic typically disappear by 3 months of age. C) Providing juice at the start of the fussy period will help decrease the length of the crying episodes. D) The feeding method needs to be changed to a hypoallergenic formula.

B

The parents of a 5-month-old child complain to the nurse that they are exhausted because the infant still wakes up as often as every 1 to 2 hours during the night. When the child awakens, they change the diaper and the mother nurses the child back to sleep. Which should the nurse suggest to help the parents deal with this problem? A) Put the child in the parents' bed to cuddle. B) Start putting the infant to bed while still awake. C) Allow the infant to cry for 30 minutes, and then rock the infant back to sleep before putting the infant back in the crib. D) Give the infant a bottle of formula instead of breastfeeding so often at night.

B

What is described as the time interval between early manifestations of a disease and the overt clinical syndrome? A) Desquamation period B) Prodromal period C) Period of communicability D) Incubation period

B

What is the causative agent for erythema infectiosum (fifth disease)? A) Paramyxovirus B) Human parvovirus B19 C) Human herpesvirus type 6 D) Group A β-hemolytic streptococcus

B

Which intervention lowers the risk of sudden infant death syndrome (SIDS)? A) Keeping the window open if one is smoking near the infant. B) Placing the infant in the supine position for sleeping. C) Letting the infant sleep with the parents instead of alone in the crib. D) Making certain the infant is kept very warm while sleeping.

B

A 1-day-old breastfed neonate, with the same blood type and Rh type as its mother, has a yellowish facial tone and a bilirubin level of 9.0. The most appropriate response by the nurse is A. "Infants' livers can't produce enough glycogen to bind to the circulating bilirubin." B. "Your baby is breaking down fetal red blood cells, and bilirubin is a waste product." C. "Your baby probably has breast milk jaundice, which is common for a 2-day-old infant." D. "We should restrict fluids until the baby can clear the bilirubin from its system."

B. "Your baby is breaking down fetal red blood cells, and bilirubin is a waste product."

In reviewing the child's family history, documentation indicates that the child is living in a household in which the two adult caregivers are not married but still continue to care for the child. This represents which type of family structure? A. Communal B. Binuclear C. Polygamous D. Nuclear

B. Binuclear

A nurse is performing a family assessment and determines that the family unit structure is composed of a father, mother, step-father, two children, a boy and a girl, and the maternal grandparents all living in the same residence. Based on this information, the nurse would indicate which family type in the electronic health record? A. Nuclear and extended B. Blended and extended C. Extended D. Blended

B. Blended and extended

Which birthmark finding would pose a concern for parents of a newborn? A. Mongolian spots B. Café-au-lait spots C. Telangiectatic nevi D. Bathing trunk nevus E. Strawberry hemangiomas

B. Café-au-lait spots D. Bathing trunk nevus

7. A camp nurse is assessing a group of children attending summer camp. Based on the nurse's knowledge of special parenting situations, which group of children is at risk for a sense of belonging? A. Children adopted as infants B. Children recently placed in foster care C. Children whose parents recently divorced D. Children who recently gained a step-parent

B. Children recently placed in foster care

The nurse is assessing newborns during a routine shift assessment. Which infant requires further assessment? A. Heart rate of 120 beats/min; respiratory rate of 45 breaths/min B. Heart rate of 135 beats/min; respiratory rate of 65 breaths/min C. Heart rate of 150 beats/min; respiratory rate of 35 breaths/min D. Heart rate of 145 beats/min; respiratory rate of 55 breaths/min

B. Heart rate of 135 beats/min; respiratory rate of 65 breaths/min

3. Studies of families with only one child indicate that only children A. tend to be selfish. B. are similar to firstborn children. C. are less stimulated toward achievement. D. grow up lonely and dependent on adults.

B. are similar to firstborn children.

8. A 7-year-old child tells the nurse, "Grandpa, Mommy, Daddy, and my sister live at my house." Based on the nurse's knowledge of family structure and function, the nurse identifies this family structure as a A. binuclear family. B. extended family. C. reconstituted family. D. traditional nuclear family.

B. extended family.

A nurse notes that the Moro reflex presents asymmetrically in the newborn and that the newborn is reluctant to move his arm on one side. Based on this finding, the nurse suspects that the newborn may have? A. congenital hip dysplasia. B. fractured clavicle. C. caput succedaneum. D. Bell's palsy.

B. fractured clavicle.

The practice of cultural humility is continual and an important concept in the nursing process. Nurses can facilitate this process by (Select all that apply.) A. focusing on their own beliefs and practices. B. recognizing cultural differences. C. integrating cultural knowledge. D. helping the family adapt to the health care practices. E. acting in a culturally appropriate manner.

B. recognizing cultural differences. C. integrating cultural knowledge. E. acting in a culturally appropriate manner.

An infant at 36 weeks' gestation with a maternal history of gestational diabetes is moved to the NICU from the transitional nursery at 3 hours of life. The infant is noted to have poor muscle tone and rapid respirations with retractions. The most probable diagnosis for this infant is? A. hyperbilirubinemia. B. respiratory distress syndrome. C. acrocyanosis. D. polycythemia.

B. respiratory distress syndrome.

The nurse is working with the parents of a young pediatric patient for which it is noted that the child is adopted. The parents ask the nurse when would be the best time to discuss the child's adoption status with the child. The nurse suggests that A. when the child reaches the age of consent. B. to begin the dialogue at an early age when the time seems right to bring up the subject. C. to wait until the child asks about his birth status. D. to refrain from bringing up the subject for fear of isolation and distrust developing.

B. to begin the dialogue at an early age when the time seems right to bring up the subject.

Which observations indicate that the neonate is experiencing jitteriness rather than a seizure? (Select all that apply.) A. present during the first week of life. B. tremors are evident C. no evidence of ocular movements present D. clonic rhythmic pattern is present E. no change in movement with stimulation

B. tremors are evident C. no evidence of ocular movements present

Which observation is associated with nightmares as opposed to sleep terrors? A: Child has no memory of the event or dream like state. B: Has a hard time returning to sleep following the event. C: Thrashing type behaviors continue when the child awakens. D: Is not comforted by traditional methods of contact.

B: Has a hard time returning to sleep following the event. Nightmares are associated with difficulty returning to sleep as opposed to sleep terrors where the individual easily goes back to sleep. With the advent of a nightmare, the child has a memory of the dream like state, is comforted by traditional methods of contact and thrashing type behaviors cease upon awakening. In sleep terrors, the child has no memory of the event, continues thrashing behaviors when awaken, and is not comforted by traditional methods of contact.

A 4-year-old child is seen playing with his father while waiting in the clinic area for a well checkup visit. The nurse observing the interaction notes that this behavior as being an example of? A: Imaginative play B: Mutual play C: Dramatic play D: Avoidance play

B: Mutual play Children playing with a parent is an example of mutual play. Play can be described as being imaginative or dramatic as a characteristic but there is no specific information given that defines the type of play interaction that is occurring. The concept avoidance play does not exist as a characteristic term describing play behaviors.

When preparing parents to teach their preschool child about human sexuality, what should the nurse emphasize? A: A parent's words may have a greater influence on the child's understanding than the parent's actions. B: Parents should determine exactly what the child wants to know before answering a question about sexuality. C: Parents should avoid using correct anatomic terms because they are confusing to the preschooler. D: Parents should encourage preschoolers to satisfy their sexual curiosity by playing "doctor."

B: Parents should determine exactly what the child wants to know before answering a question about sexuality. It is important that the parent answer the question that the child is asking. Actions may have a greater influence because language is not fully developed in the preschool years. Using correct terminology lays the foundation for later discussion of human sexuality. Parents should encourage children to ask questions to provide accurate information at their cognitive level.

A mother tells the nurse that her daughter's favorite toy is a large, empty box that contained a stove. She plays "house" in it with her toddler brother. Based on the nurse's knowledge of growth and development, the nurse recognizes that this is: A: unsafe play that should be discouraged. B: creative play that should be encouraged. C: suggestive of limited family resources. D: suggestive of limited adult supervision.

B: creative play that should be encouraged. This type of play should be encouraged. After children create something new, they can then transfer it to other situations. There should be some supervision to prevent injury or accidents. As long as the play is supervised, it should be encouraged. This is not considered unsafe play. There is no indication of limited resources. There is no indication of limited adult supervision.

The best approach for effective communication with a preschooler is through A: speech. B: play. C: drawing. D: actions.

B: play. Preschoolers' most effective means of communication is through play. Play allows preschoolers to understand, adjust to, and work out life's experiences through their imagination and ability to invent and imitate. Speech is not effective, because preschoolers assume that everyone thinks as they do and that a brief explanation of their thinking makes them understood by others, which is often not true. Also, preschoolers often do not understand the meaning of words and often take statements literally. Drawing is still being developed as a fine motor skill; therefore, it is not the most effective means of communication. Actions are not an appropriate means of communication for a preschooler.

A child is admitted to the hospital with a diagnosis of possible meningitis. Which information is the most important to ask at the time of admission? A) "Is anyone else in the household ill?" B) "Are there any pets in the household?" C) "Are the immunizations up to date?" D) "Has the child had a recent injury?"

C

A nurse has been assigned to take care of a pediatric patient suspected of having chicken pox(varicella). Which type of precaution is required? A) Droplet precautions B) Contact precautions C) Airborne with isolation room D) Standard precautions are sufficient

C

A nurse is providing education to a community group in preparation for a mission trip to a third world country with limited access to protein-based food sources. The nurse is aware that children in this country are at increased risk for A) rickets. B) marasmus. C) kwashiorkor. D) pellagra.

C

According to evidenced based practice in a recent Cochrane database, which statement is most accurate with regard to the relationship between vitamin A and measles? A) It is correlated with an increased risk of blindness. B) Vitamin A supplementation should not be provided to children who have measles. C) There was no correlation between blindness in children who had measles and who also had vitamin A deficiency. D) Vitamin A supplementation in children with measles will lead to blindness.

C

In working with a group of parents related to providing information relative to car seat restraints. Which statement by a member of the parent group would indicate that additional teaching was necessary? A) "I will place the car seat facing backwards in the car." B) "The car seat should be anchored to the vehicle's seat belt." C) "The LATCH system should be used for a child who is below 35 pounds." D) "The child should be in a rear facing position up to 24 months of age."

C

Nurses play an important role in current issues and trends in health care. Which is a current trend in pediatric nursing and health care today? A) The focus of pediatric health care is trending toward acute hospital care. B) The patient is the unit of care for the health care provider. C) Health promotion resources enable children to achieve their full potential. D) Discharge planning begins when the physician writes the order.

C

Parents of an infant report that the child is extremely fuzzy at times. Which parental response might lead to potential poor outcomes in later life for the child? A) Attempts to provide comfort by playing music. B) Holding the infant while applying a gentle rocking motion. C) Offering the infant more food to decrease fuzziness. D) Playing with the infant to attempt distraction

C

The nurse demonstrates understanding of family-centered care by A) expecting the child to perform self-care in activities of daily living. B) assuming total care for the child. C) encouraging family visitation. D) limiting visitation to three time periods per day.

C

The nurse is assessing a child with herpetic gingivostomatitis. The nurse wears gloves when examining the lesions. This nursing action is A) necessary only if the nurse touches his or her own mouth after touching the child's mouth. B) unnecessary because the virus is not easily spread. C) necessary because virus can easily enter breaks in the skin. D) unnecessary because the virus is sexually transmitted.

C

The parents of a 9-month-old infant tell the nurse that they are worried about their baby's thumb sucking. What should the nurse's reply be based on? A) A pacifier should be substituted for the thumb. B) Thumb sucking should be discouraged by age 12 months. C) Thumb sucking should be discouraged when permanent teeth begin to erupt. D) There is no need to restrain nonnutritive sucking during infancy.

C

The role of the pediatric nurse is influenced by trends in health care. Which is an influential trend in pediatric health care based on Healthy People 2030 goals? A) Primary focus on the pediatric age range. B) Focusing educational materials for the bilingual population. C) Engaging key partnerships in design of healthcare policies. D) Focus on the unification of the population to meet one common standard.

C

Which assessment finding would the nurse expect to see with regard to weight status in an infant who is 1 year of age? A) Weight is doubled that of birth weight. B) Average weight of a 1 year old is stable if found to be 15 pounds. C) Weight is tripled that of birth weight. D) Breastfeeding infants typically will have smaller head circumference than bottle fed infants. E) Weight is quadrupled compared with initial birth weight at age one.

C

Which infant is at risk for developing vitamin D-deficient rickets? A) Lacto-ovo vegetarians B) Those who are breastfed exclusively C) Those using yogurt as a primary source of milk D) Those exposed to daily sunlight

C

Which is the most appropriate recommendation for relief of teething pain? A) Rub the gums with aspirin to relieve inflammation. B) Apply hydrogen peroxide to the gums to relieve irritation. C) Give the child a frozen teething ring to relieve inflammation. D) Have the child chew on a warm teething ring to encourage tooth eruption.

C

Which represents the most likely cause of death for children greater than 1 year of age? A) Violent death B) Homicide C) Unintentional injuries D) Congenital diseases

C

Which statement best describes colic to parents who are inquiring as to whether their infant is experiencing this alteration? A) The infant will experience periods of abdominal pain, which result in weight loss. B) Periods of abdominal pain and crying occur in infants primarily over age 6 months. C) Infants with colic have paroxysmal abdominal pain or cramping manifested by episodes of loud crying. D) Colic is usually the result of poor or inadequate mothering.

C

Which strategy might be recommended to increase caloric intake in an infant with failure to thrive? A) Use developmental stimulation by a specialist during feedings. B) Avoid solids until after the bottle is well accepted. C) Be persistent through 10 to 15 minutes of food refusal. D) Vary the schedule for routine activities on a daily basis.

C

Which delivery situations might possibly result in soft tissue damage to the head of a newborn? Select all that apply. A. Spontaneous vaginal delivery. B. Administration of an epidural for pain relief. C. Dystocia as a result of cephalopelvic disproportion (CPD). D. Use of forceps. E. Use of vacuum suction cap.

C. Dystocia as a result of cephalopelvic disproportion (CPD). D. Use of forceps. E. Use of vacuum suction cap.

4. Which is appropriate advice for parents who are preparing to tell their children about their decision to divorce? A. Avoid crying in front of children. B. Avoid discussing the reason for the divorce. C. Give reassurance that the divorce is not the children's fault. D. Give reassurance that the divorce will not affect most aspects of the children's life.

C. Give reassurance that the divorce is not the children's fault.

The nurse receives a report about a newborn who was resuscitated at birth secondary to poor respiratory effort. Which assessment data would concern the nurse 6 hours after birth? A. Sleepiness during feeding B. Acrocyanotic hands and feet C. Low body temperature D. Respirations of 40 breaths/min

C. Low body temperature

In consideration of family systems theory, which components form the triangle relationship? (Select all that apply.) A. Grandparents B. Cousin C. Mother D. Child E. Father

C. Mother D. Child E. Father

Which clinical diagnosis is associated with an increased risk for the newborn to develop hypovolemic shock? A. Caput succedaneum B Cephalohematoma C. Subgaleal hemorrhage D. Bleeding into the area between the bone and periosteum E. Spontaneous vaginal delivery

C. Subgaleal hemorrhage

9. Successful adaptation to the stress of transition to parenthood involves two types of family resources. These resources include (Select all that apply) A. adaptation. B. integration. C. coping strategies. D. internal resources E. community resources.

C. coping strategies. D. internal resources

A nurse observes a parent-child interaction in which the parent acts indifferently to the child's continued efforts to play with a water cooler in the clinic's waiting room resulting in a small amount of flooding in the area of the room. Based on this assessment, the nurse would determine that the parent's childrearing style would be noted as A. authoritative. B. directive. C. permissive. D. authoritarian.

C. permissive.

The nurse is giving anticipatory guidance to the parent of a 5-year-old. What is the most appropriate information to include? A: Prepare the parent for increased aggression. B: Encourage the parent to offer the child choices. C: Inform the parent that he or she can expect a more tranquil period at this age. D: Advise the parent that this is the age when stuttering may develop.

C: Inform the parent that he or she can expect a more tranquil period at this age. The end of preschool and the beginning of school age is a more tranquil period. Preparing the parent for increased aggression is anticipatory guidance for 4-year-old children. Encouraging the parent to offer the child choices is anticipatory guidance for 3-year-old children. Advising the parent that this is the age when stuttering may develop is anticipatory guidance for 3-year-old children.

The preschooler's body image has developed to include: A: a well-defined body boundary. B: knowledge about his or her internal anatomy. C: fear of intrusive procedures. D: fear of looking different from his or her friends.

C: fear of intrusive procedures. Preschoolers fear that their insides will come out with intrusive procedures. Preschoolers have poorly defined body images. Preschoolers have little or no knowledge of their internal anatomy. The fear of looking different is a concept that occurs in later school-aged children and adolescents.

The two rules used to provide sensitive information to a preschooler focus on: A: describing the information in a succinct way and answering any questions. B: ask the child if he/she understands what you have told them and then clarify any additional questions. C: find out what they know and think and then be honest. D: use the opportunity to provide minimal information and keep the level of interaction simple.

C: find out what they know and think and then be honest. It is important to find out what they know and think before providing any information and then equally important is to be honest in your communication. The method of providing information may vary based on the type of information being provided and the level of understanding of the child.

A nurse has been working with a family who has 2 children, ages 5 and 7 years of age, to provide health teaching related to healthy nutritional patterns. Which observation if noted would indicate that additional instruction should be given related to health teaching in this area? A) The children demonstrate application of skills by identifying healthy food snacks. B) The nurse provides an interactive learning environment using age appropriate learning strategies. C) The parents are able to identify which foods are poor examples of "healthy food snacks." D) In response to the family's identification of healthy food snacks, the nurse provides limited feedback since the answers are correct.

D

A nurse is caring for a 2-month-old exclusively breastfed infant with an admitting diagnosis of colic. Based on the nurse's knowledge of breastfed infants, what type of stool is expected? A) Dark brown and small hard pebbles B) Loose with green mucus streaks C) Formed and with white mucus D) Semiformed, seedy, yellow

D

A nurse is providing instruction to a family concerning the care of an infant with bacterial conjunctivitis. Which observation indicates that additional teaching is needed? A) Separate towels are being used for the infant. B) Warm, moist compresses are being used to cleanse the eye area. C) Tissues used to clean the eye are discarded following use. D) An occlusive dressing is applied to the eye area.

D

An infant is more likely to be at increased risk for infections based on the immunological premise that A) decreased amount of immunoglobulin M at birth. B) limited maternal transfer resulted in decreased protection during the first 3 months of life. C) inability to synthesize immunoglobulin G. D) limited ability to reach adult levels until 1 year of age.

D

In providing nutritional counseling for a family with children, which statement would indicate that the parents need additional teaching with regard to mineral balance? A) "I will give my child fortified milk products and avoid cow's milk." B) "I will avoid giving my children any mineral supplements so as to avoid the possibility of megadoses." C) "Spinach is not a very source of iron when considering mineral balance." D) "I don't have to worry about mineral balance since my child will be following a vegetarian diet."

D

The etiology component of the nursing diagnosis describes the A) cluster of cues and/or defining characteristics that are derived from patient assessment and indicate actual health problems. B) individual's responses to health pattern deficits in the child, family, or community. C) projected changes in an individual's health status, clinical conditions, or behavior. D) physiologic, situational, and maturational factors that cause the problem or influence its development.

D

The infant's mother reports that her 6-month-old daughter seems to be afraid of strangers now. Based on this reported finding the nurse would advise the mother that A) psychological counseling may be needed if the behavior continues or worsens. B) this is probably due to "nightmares" and should be self-limiting, so there is no need to worry. C) ask the mother what type of specific behaviors was the child exhibiting as there may be potential abuse occurring. D) tell the mother that fear of strangers is normal during this time period and typically will be self-limiting.

D

The nurse is providing education to a group of parents at a health fair in a local kindergarten school. The nurse describes the most common cause of death for children age 5 to 9 years is A) lack of hand washing in the prevention of communicable diseases. B) the obesity epidemic. C) childhood immunizations. D) inappropriate use of bike helmets and seat belts.

D

Which nursing intervention would be most effective in decreasing mortality from unintentional injury? A) Encouraging potential parents to obtain genetic counseling B) Teaching children the dangers of contact sports C) Educating parents-caretakers about the benefits of immunizations D) Teaching parents-caretakers about proper use of vehicle restraint seats

D

Which recommendation would a nurse make to new parents who are planning to introduce solid foods to their 6-month-old son? A) Cream of wheat cereal B) Cream of farina cereal C) Cereals fortified with iron D) Rice cereal

D

6. Research notes that birth position of children affects their personalities. According to ordinal position, what is a characteristic of the youngest child? A. Able to identify more with their parents than with their peers B. Are expected to do more household chores C. More dependent than firstborn children D. Are more flexible in their thinking

D. Are more flexible in their thinking

2. Which phrase is descriptive of homosexual or gay-lesbian families? A. Nurturing environment is lacking B. Stability needed to raise healthy children is lacking C. Sexual identities of children are at risk D. Family environment can be just as healthy as any other

D. Family environment can be just as healthy as any other

With regard to the varied definitions of the term, family, which variable would best represent the psychological definition? A. Productivity of its members B. Focus on childbearing attributes C. Consanguineous relationships D. Interpersonal skill development

D. Interpersonal skill development

The nurse is gavage feeding a preterm neonate. Which nursing intervention should be introduced prior to and during the gavage feeding? A. Position the neonate on the abdomen between feedings B. Provide blow-by oxygen prior to feedings C. Warm the gavage tube prior to insertion D. Provide a pacifier during feedings

D. Provide a pacifier during feedings

The nurse in the delivery room should anticipate that which of the following measures will be taken first for an infant delivered with meconium-stained fluid? A. Bag-and-mask ventilations B. Immediate intubation after delivery C. Oxygen applied by mask D. Use of a bulb syringe to suction the mouth and nose before the shoulders are delivered

D. Use of a bulb syringe to suction the mouth and nose before the shoulders are delivered

1. Which is descriptive of family system theory? A. Family is viewed as the sum of individual members. B. Change in one family member cannot create a change in other members. C. Individual family members are readily identified as the source of a problem. D. When the family system is disrupted, change can occur at any point in the system.

D. When the family system is disrupted, change can occur at any point in the system.

The nurse is planning care for a patient with a different ethnic background from the nurse's own. The most appropriate goal for the nurse in caring for this patient is to A. attempt, in a nonjudgmental way, to change ethnic beliefs. B. strive to keep ethnic background from influencing health care. C. encourage continuation of ethnic practices in the hospital setting. D. adapt, as necessary, ethnic practices to health needs.

D. adapt, as necessary, ethnic practices to health needs.

5. A nurse is assessing a family for effective coping and defensive strategies. The family social system theory the nurse will use is the A. family systems theory, as derived from general systems theory. B. resiliency Model of Family Stress, Adjustment, and Adaptation. C. family developmental theory. D. family stress theory.

D. family stress theory.

A newborn is delivered at 33 weeks' gestation and is admitted to the special care nursery after administration of exogenous surfactant in the delivery room. The diagnosis associated with the use of surfactant is? A. meconium aspiration syndrome. B. apnea of prematurity. C. persistent pulmonary hypertension. D. respiratory distress syndrome.

D. respiratory distress syndrome.

Parents of a preschooler relate that their child is having some difficulties falling asleep. This is a recent occurrence and the parents report that they have tried virtually everything to get their child to go to sleep. Based on this self-report, the nurse would advise the parents to? A: Do not feed the child 2 hours before sleep as food might be causing excess stimulation. B: Do not let the child watch any television in the evening as this may cause stimulation. C: Have the child fall asleep on the couch and then transfer the child to the bed. D: Maintaining a consistent approach to bedtime routine serves as a basis for promoting sleep patterns.

D: Maintaining a consistent approach to bedtime routine serves as a basis for promoting sleep patterns. Children in this age group often experience changes in their sleep patterns but consistent approaches in bedtime rituals may help to promote sleep patterns. By the parents self-report that they have "tried virtually everything" may have contributed to the child's altered sleep pattern. There is no enough information provided for the nurse to make any other of the provided options.

Which physical assessment findings would be associated with the presence of alopecia? Select all that apply. Excess vitamin C Decreased protein intake Decreased caloric intake Decreased copper Decreased zinc

Decreased protein intake Decreased caloric intake Decreased zinc The presence of alopecia could be related to decreases in protein and caloric intake. It is also associated with decreased intake of zinc. Decreased intake of zinc is associated with depigmentation. Excess vitamin C can be associated with pallor.

What is the most appropriate nursing intervention when caring for a child experiencing a seizure?

Describe and document the seizure activity observed.

What are signs of shunt malformation in a child who had a shunt revision for hydrocephaly?

Dizziness, Bulging anterior fontanel.

What statement should the nurse include when discussing a child's precocious puberty with the parents?

Dress and activities should be appropriate to the chronologic age.

A pediatric patient is brought to the clinic for follow up immunizations and is apprehensive about injections and needles. Which method could the nurse implement to facilitate administration of immunization without causing the child undue anxiety? Offer the child an ice cream if you are allowed to give the injection. Tell the patient's mother that she must swaddle him and cover his eyes tightly once the medication is drawn up and ready to be injected. Encourage the child to talk about the toy he is holding and focus on the conversation to allay the child's fears. Instead of an injection, provide the medication via a topical route.

Encourage the child to talk about the toy he is holding and focus on the conversation to allay the child's fears. The use of distraction may be an effective method in allaying fears related to needles and injections. Offering a "reward" for the action is not an effective method for dealing with anxiety about medication administration. Swaddling the child in combination with covering his eyes is not an appropriate way to decrease apprehension and may escalate anxiety. The route of administration for an immunization is not changed and is solely based on established guidelines.

The nurse is discussing toddler development with a parent. Which intervention will foster the achievement of autonomy? Help the toddler complete tasks. Provide opportunities for the toddler to play with other children. Help the toddler learn the difference between right and wrong. Encourage the toddler to do things for himself or herself when he or she is capable of doing them.

Encourage the toddler to do things for himself or herself when he or she is capable of doing them. Toddlers have an increased ability to control their bodies, themselves, and the environment. Autonomy develops when children complete tasks of which they are capable. To successfully achieve autonomy, the toddler needs to have a sense of accomplishment. This does not occur if parents complete tasks. Children at this age engage in parallel play. This will not foster autonomy. This concept is too advanced for toddlers and will not contribute to autonomy.

Which characterizes the development of a 2-year-old child? Engages in parallel play Fully dresses self with supervision Has a vocabulary of at least 500 words Has attained one third of his or her adult height

Engages in parallel play Two-year-olds play alongside each other, otherwise known as parallel play. Toddlers need help with dressing because this is a task they are just beginning to learn; learning this extends into the preschool years. A toddler commonly has a vocabulary of 300 words. A toddler has attained one half of his or her adult height.

Psychosocial Development: Erikson's Stage During Preschool

Erikson maintained that the chief psychosocial task of the period during preschool is acquiring a sense of initiative. Children are in a stage of energetic learning. They play, work, and live to the fullest and feel a real sense of accomplishment and satisfaction in their activities. Conflict arises when children overstep the limits of their ability and inquiry and experience a sense of guilt for not having behaved appropriately. Feelings of guilt, anxiety, and fear may also result from thoughts that differ from expected behavior.

Which observable behaviors would indicate to the nurse that the patient is experiencing information overload? Select all that apply. Fidgeting constantly while seated in the chair A period of silence noted between a question The patient wanting to continue talking about one subject of interest The patient is yawning repeatedly The patient is scanning the environment avoiding eye contact while the nurse is attempting to ask questions.

Fidgeting constantly while seated in the chair The patient is yawning repeatedly The patient is scanning the environment avoiding eye contact while the nurse is attempting to ask questions. Examples of information include but are not limited to: constant fidgeting, repetitive yawning, and avoidance of eye contact while looking away from the interviewer and scanning the environment. A period of silence noted between one question is not in itself indicative of information overload unless the silence continues. The patient wanting to focus on one subject of interest is not associated with information overload.

What should the nurse recommend to the parents to help a toddler cope with the birth of a new sibling? Give the toddler a doll with which he or she can imitate the parents. Discourage the toddler from helping with care of the new sibling until the baby is much older. Prepare the toddler about 1 to 2 weeks before the birth of a new sibling. Explain to the toddler that a new playmate will soon come home.

Give the toddler a doll with which he or she can imitate the parents. The toddler can participate in the activity of caring for a new family member, which will make him or her feel included and important. The child should be encouraged to participate within his or her capabilities. The toddler should never be discouraged, because this will make him or her feel isolated and left out. Preparation should begin when obvious changes begin to happen to the mother's body and at home. This will establish unrealistic expectations for the toddler. Toddlers take language literally, and therefore will be disappointed when the new baby cannot play when he or she arrives home.

Which statement about bottle-mouth caries should be taught to the parents? This syndrome is distinguished by protruding upper front teeth, resulting from sucking on a hard nipple. Giving a bottle of milk or juice at nap time or bedtime predisposes the child to this syndrome. This syndrome can be completely prevented by breastfeeding. Giving the child juice in the bottle instead of milk at bedtime prevents this syndrome.

Giving a bottle of milk or juice at nap time or bedtime predisposes the child to this syndrome. Sweet liquids, or the sugars in milk and even breast milk, pooling in a toddler's mouth during sleep increase the incidence of dental caries. Changes in the positioning of the teeth may result from pacifier use or thumb sucking and are not related to bottle-mouth caries. Frequent breastfeeding before sleep can cause bottle-mouth caries, since breast milk does contain lactose, which is present in higher concentrations than in cow's milk-based formula. Juice, which contains varying concentrations of sugar, in bottles before sleep contributes to bottle-mouth caries.

Which of the following statements is an accurate representation of the Apgar scoring system? It is predictive of infant morbidity. It is an evaluation of newborn well-being. It determines the predictive growth pattern in the first month of life. It determines the diagnosis of cerebral palsy.

It is an evaluation of newborn well-being. Apgar scoring is a reflection of newborn well-being and includes evaluation of heart rate, respiratory effort, muscle tone, reflex irritability, and color.

What diagnostic tests should be included when assessing a child for bacterial meningitis?

Kernig's sign, Romberg test.

6. A nurse is discussing various developmental theories at a parenting class. Which individual is associated with the moral development theory? • Erikson • Fowler • Kohlberg • Freud

Kohlberg

Which of the following observations suggests that an abnormality may be present in a full-term neonate? Absence of tears Engorged breasts Lack of a sucking reflex Inability to visually fix and follow object

Lack of a sucking reflex

Which type of disease presentations are associated with recurrent pain in children? Select all that apply. Daily headache Migraine headache Abdominal pain Complex regional pain syndrome Phantom limb pain

Migraine headache Abdominal pain Phantom limb pain Common presentations of recurrent pain in children include but are not limited to: migraine headaches, abdominal pain, and phantom limb pain. Daily headache and complex regional pain syndrome are categorized as chronic pain in children.

A toddler is admitted to the emergency room with a possible diagnosis of accidental poisoning. Which information if noted on the admission form would indicate a potential cause for this clinical diagnosis? Parents do not take any prescribed medications. Cleaning supplies are kept in locked cabinets in the child's home. Mother has been taking Tylenol OTC for sinus headaches for several days and keeps medication in a tote handbag Toddler has just finished treatment with oral antibiotics for ear infection 1 week ago.

Mother has been taking Tylenol OTC for sinus headaches for several days and keeps medication in a tote handbag The fact that the Tylenol is being kept in a handbag may pose concern as access to the tote handbag is not restrictive. The toddler may have been able to access the medication. The other options do not pose a significant concern as a possible etiology for this clinical diagnosis.

Children's Responses to Pain at Various Ages

Newborn and Young Infant • Uses crying • Reveals facial appearance of pain (brows lowered and drawn together, eyes tightly closed, and mouth open and squarish) • Exhibits generalized body response of rigidity or thrashing, possibly with local reflex withdrawal from what is causing the pain • Shows no relationship between what is causing the pain and subsequent response • Newborn and infant consistent indicator of pain ○ Fussiness, crying, and grimacing Older Infant • Uses crying • Shows a localized body response with deliberate withdrawal from what is causing the pain • Reveals expression of pain or anger • Demonstrates a physical struggle, especially pushing away from what is causing the pain Young Child • Uses crying and screaming • Uses verbal expressions, such as "Ow," "Ouch," or "It hurts" • Uses thrashing of arms and legs to combat pain • Attempts to push what is causing the pain away before it is applied • Displays lack of cooperation; need for physical restraint • Begs for the procedure to end • Clings to parent, nurse, or other significant person • Requests physical comfort, such as hugs or other forms of emotional support • Becomes restless and irritable with ongoing pain • Worries about the anticipation of the actual painful procedure School-Age Child • Demonstrates behaviors of the young child, especially during actual painful procedure, but less before the procedure • Exhibits time-wasting behavior, such as "Wait a minute" or "I'm not ready" • Displays muscular rigidity, such as clenched fists, white knuckles, gritted teeth, contracted limbs, body stiffness, closed eyes, wrinkled forehead Adolescent • Less vocal with less physical resistance • More verbal in expressions, such as "It hurts" or "You're hurting me" • Displ

Thermoregulation

Next to establishing respiration, heat regulation is most critical to the newborn's survival. Although the newborn's capacity for heat production is adequate, three factors predispose newborns to excessive heat loss: • The newborn's large surface area facilitates heat loss to the environment, although this is partially compensated for by the newborn's usual position of flexion, which decreases the amount of surface area exposed to the environment. • The newborn's thin layer of subcutaneous fat provides poor insulation for conservation of heat. • The newborn's mechanism for producing heat is different from that of the adult, who can increase heat production through shivering. A chilled neonate cannot shiver but produces heat through nonshivering thermogenesis (NST), which involves increased metabolism and oxygen consumption. -The principal thermogenic sources are the heart, liver, and brain. An additional source, once believed to be unique to newborns, is known as brown adipose tissue, or brown fat. Brown fat, which owes its name to its larger content of mitochondrial cytochromes, has a greater capacity for heat production through intensified metabolic activity than ordinary adipose tissue. Heat generated in brown fat is distributed to other parts of the body by the blood, which is warmed as it flows through the layers of this tissue.

What should the nurse do if a toddler who had brain tumor surgery becomes irritable with unequal and sluggish pupils?

Notify the practitioner immediately.

What is true about osteogenesis imperfecta (OI)?

OI is an inherited disorder.

What priority assessments should be included in the plan of care for a child with submersion injury?

Obtain arterial blood gases (ABGs), Maintain airway and ventilator support.

9. Which behavioral pattern would be a cause for concern to a nurse for in a pediatric male patient, 8 years of age, who is presenting to the clinic with his parents for a well-child visit? • Child is quiet playing with his iPad while his parents answer questions posed by the nurse. • Parents are laughing and joking with their son regarding an earlier event that occurred that day. • Parents are telling their son that he is going to get fat if he continues to keep eating pretzels before dinner. • Child asks to borrow the nurse's stethoscope to see how it works.

Parents are telling their son that he is going to get fat if he continues to keep eating pretzels before dinner.

Pediculosis Capitis

Pediculosis capitis (head lice) ● Nursing Care Management ○ Can be transmitted from person to person ○ Carefully inspect children who scratch their heads more than usual for bite marks, redness, and nits ○ Permethrin 1% lotion or shampoo is the first line of treatment ○ Carefully comb out all nits with a comb

10. A nurse is assessing parental styles of a family and notes little to no control over their children. This style of parenting is called __________________________.

Permissive

What is the term for pain in a missing extremity after amputation?

Phantom limb sensation.

Guillain-Barre Syndrome

Priority Care: ○ Administration of immunoglobulin (IVIG) is now recommended as the primary treatment of GBS when administered within 2 weeks of disease onset ○ Close observation to assess the extent of paralysis and on prevention of complications, including aspiration, ventilator-associated pneumonia, atelectasis, DVT, pressure ulcer, fear and anxiety, autonomic dysfunction, and pain. ○ Respiratory function is closely monitored

Which measure is important in managing hypercalcemia in a child who is immobilized?

Promote adequate hydration.

Which statement is true regarding the World Health Organization (WHO) principles of pharmacologic management for the pediatric patient? Select all that apply. Provide a step-ladder approach Provide dosing at regular intervals Choice of analgesic as the first step with administration of morphine as the second step Using parenteral route for pain therapies Individualizing and adapting treatment for each child

Provide dosing at regular intervals Choice of analgesic as the first step with administration of morphine as the second step Individualizing and adapting treatment for each child WHO has gone from a step-ladder approach to a twostep approach for pain management therapy for children beginning with a choice of analgesic with a narcotic opioid, typically morphine is the second step. Dosing is recommended at regular intervals with adapting treatment to the individual child. Appropriate route for medication is recommended based on clinical pharmacology.

What is the priority assessment for a child with variable level of consciousness after sustaining a cervical neck injury?

Reactivity of pupils

What is the most accurate method of determining the length of a child younger than 12 months of age? Standing height Estimation of length to the nearest centimeter or 1/2 inch Recumbent length measured in the prone position Recumbent length measured in the supine position

Recumbent length measured in the supine position The crown-heel length measurement is the most accurate measurement in infants. Infants are generally unable to stand for obtaining a height measurement. Measurement should not be estimated, because an accurate measurement is required to determine growth. The infant should be measured in the supine position, not the prone position.

A nurse is assessing a pediatric patient in the intensive care unit. Which finding on the COMFORT score if noted would not require intervention relative to pain management? Score of 8 Score of 10 Score of 25 Score of 40

Score of 25 Scores ranging between 17 and 26 typically indicate adequate pain control. Total scale score range between 8 and 40.

Hospitalized Children

Separation Anxiety -Stage of Protest: Behaviors observed during later infancy include: • Cries • Screams • Searches for parent with eyes • Clings to parent • Avoids and rejects contact with strangers Additional behaviors observed during toddlerhood include: • Verbally attacks strangers (e.g., "Go away") • Physically attacks strangers (e.g., kicks, bites, hits, pinches) • Attempts to escape to find parent • Attempts to physically force parent to stay Behaviors may last from hours to days. Protest, such as crying, may be continuous, ceasing only with physical exhaustion. Approach of a stranger may precipitate increased protest. Stage of Despair: Observed behaviors include: • Is inactive • Withdraws from others • Is depressed, sad • Lacks interest in environment • Is uncommunicative • Regresses to earlier behavior (e.g., thumb sucking, bedwetting, use of pacifier, use of bottle) Behaviors may last for variable length of time. Child's physical condition may deteriorate from refusal to eat, drink, or move. Stage of Detachment: Observed behaviors include: • Shows increased interest in surroundings • Interacts with strangers or familiar caregivers • Forms new but superficial relationships • Appears happy Detachment usually occurs after prolonged separation from parent; it is rarely seen in hospitalized children. Behaviors represent a superficial adjustment to loss. Sibling reactions: Siblings experience loneliness, fear, and worry, as well as anger, resentment, jealousy, and guilt.

What laboratory finding would indicate a critical assessment in a child with Syndrome of Inappropriate Anti-Diuretic hormone (SIADH)?

Serum sodium 120 mEq/L.

What is a clinical manifestation of increased intracranial pressure (ICP) in infants?

Shrill, high-pitched cry

What is the most important part of the discussion with parents of a child with a ventriculoperitoneal shunt?

Shunt malfunction or infection requires immediate treatment.

Which of the following is the most appropriate intervention to reduce stress in a preterm infant at 33 weeks of gestation? Skin-to-skin contact with parent Sensory stimulation involving several senses at a time Tactile stimulation until signs of overstimulation develop An attitude of extension when prone or side-lying

Skin-to-skin contact with parent

What is the term for a seizure that lasts 35 minutes with loss of consciousness?

Status epilepticus.

What is a characteristic of most neonatal seizures?

Subtle and barely discernible seizure

Immunizations

The Tdap (tetanus, diphtheria, acellular pertussis) vaccine is recommended for adolescents 11 to 18 years old who have not received a tetanus-diphtheria booster (Td) or Tdap dose and have completed the childhood diphtheria, tetanus, and acellular pertussis (DTaP/DTP) series. When Tdap is used as a booster dose, it may be administered at any time earlier than the previous 5-year interval to provide adequate pertussis immunity (regardless of interval from the last Td dose). -Meningococcal vaccine (Menactra or Menveo) should be given to adolescents 11 to 12 years old with a booster dose at 16 years old. If not previously vaccinated, they should receive one dose at 13 through 18 years old. -Influenza vaccine is recommended for all children and adolescents annually.

Which of the following statements reflects accurate information about patterns of sleep and wakefulness in the infant? Cycles of sleep states are uniform in infants of the same age. The alert quiet state is the best stage for infant stimulation. States of sleep are independent of environmental stimuli. Irregular breathing is common during deep sleep.

The alert quiet state is the best stage for infant stimulation. The infant is awake and visually exploring the surroundings during the alert quiet state; thus it is the best time for stimulation.

A hospitalized toddler clings to a worn, tattered blanket. The toddler screams when anyone tries to take it away. Which is the best explanation for the toddler's attachment to the blanket? The blanket encourages immature behavior. The blanket is an important transitional object. The developmental task of individuation-separation has not been mastered. The child and mother have inadequate bonding.

The blanket is an important transitional object. The blanket is an important transitional object that provides security when the child is separated from parents. Transitional objects are important to help toddlers separate, and attachment to them does not indicate immature behavior. Transitional objects are helpful when a toddler experiences increased stress such as hospitalization. The attachment to the blanket does not reflect inadequate bonding with the mother.

When interviewing a patient, which statement/action indicates that the nurse is displaying empathy? The nurse offers the patient a tissue when the patient is crying after hearing some sad news before giving the patient medication. The nurse and patient discuss their families and discover they each have two brothers. The patient appreciates that the nurse has sat by her bedside and held her hand while they spoke about health concerns. The nurse provided the patient's family with Advanced Directive Form to fill out acknowledging that it has to be done in order to fulfill the patient's wishes.

The patient appreciates that the nurse has sat by her bedside and held her hand while they spoke about health concerns. Empathy convey that the person understands, is supportive and can view the situation from the other person's perspective whereas sympathy is the acknowledgement of another person's feelings or emotions without the context of understanding. Providing a tissue to a patient is an example of sympathy. Finding out that the nurse and patient have similar number of family members does not convey either empathy or sympathy but rather factual disclosure. The nurse providing an Advanced Directive Form is part of one's nursing role.

Which is an important consideration when using the FACES Pain Rating Scale with children? Children color the face with the color they choose to best describe their pain. The scale can be used with most children as young as 3 years of age. The scale is not appropriate for use with adolescents. The scale is useful in pain assessment but is not as accurate when assessing physiologic responses.

The scale can be used with most children as young as 3 years of age. The FACES scale has been validated for children as young as 3 years of age to rate pain. The child points at the face that best describes the pain being experienced. The scale is useful for all ages above 3 years, including adults. The scale does not have a means of assessing physiologic data.

8. Which statement helps explain the growth and development of children? • Development proceeds at a predictable rate. • The sequence of developmental milestones is predictable. • Rates of growth are consistent among children. • At times of rapid growth, there is also acceleration of development.

The sequence of developmental milestones is predictable.

Psychosocial Development: Erikson's Stage during Childhood & Adolescence

The task of identity formation is to develop a stable, coherent picture of oneself that includes integrating one's past and present experiences with a sense of where one is headed in the future. -Throughout childhood, individuals have been going through the process of identification as they concentrate on various parts of the body at specific times. -During infancy, children identify themselves as being separate from the mother; during early childhood, they establish gender role identification with the appropriate-sex parent; and in later childhood, they establish who they are in relation to others. In adolescence, they come to see themselves as distinct individuals, somehow unique and separate from every other individual. -Adolescence begins with the onset of puberty and extends to relative physical and emotional stability at or near graduation from high school. During this time, adolescents are faced with the crisis of group identity versus alienation. In the period that follows, individuals strive to attain autonomy from the family and develop a sense of personal identity as opposed to role diffusion. A sense of group identity appears to be essential to the development of a personal identity. Young adolescents must resolve questions concerning relationships with a peer group before they are able to resolve questions about who they are in relation to family and society.

Nephrotic Syndrome

Therapeutic management: ○ Reducing excretion of urinary protein ○ Reducing fluid retention of tissues ○ Preventing infection ○ Minimizing complications related to therapies

Which statement characterizes toddlers' eating behavior? They have increased appetites. They have few food preferences. Their table manners are predictable. They become fussy eaters.

They become fussy eaters. Toddlers have physiologic anorexia, which contributes to picky, fussy eating. This usually begins at about 18 months of age. They have a decrease in appetite known as physiologic anorexia at this age. They have strong taste preferences at this age. Use of finger foods contributes to the unpredictable table manners of toddlers.

Thermoregulation presents a potential problem for neonates. The primary cause of this potential thermoregulation instability would be: Their renal function is not fully developed. Their small body surface area favors heat loss. They have a thin layer of subcutaneous fat that provides poor insulation. They maintain a flexed posture that promotes heat loss.

They have a thin layer of subcutaneous fat that provides poor insulation. Infants have small amounts of subcutaneous fat, which does not provide insulation and thus prevents them from retaining heat.

Which food activities would be considered to be normal adaptations for toddlers? Select all that apply. They often pick up many types of foods, preferring snacking rather than eating just at mealtimes. Appetite and food choices exhibit a consistent pattern. Toddlers often exhibit "picky" eating behaviors which is considered to be a normal abnormal. Children tend to eat more when they are growing. Foods should not be given to children during playtime.

They often pick up many types of foods, preferring snacking rather than eating just at mealtimes. Toddlers often exhibit "picky" eating behaviors which is considered to be a normal abnormal. Foods should not be given to children during playtime. Toddlers exhibit grazing or nibbling behaviors and sometimes prefer to snack throughout the day. Toddlers often exhibit "picky" preferential behaviors in their food choices. Foods should not be given to children during playtime as a safety measure to prevent possible choking or aspiration. Appetite and food choices are inconsistent during this time frame. Toddlers will eat more during growth spurt periods.

Why are external defects of the genitourinary tract repaired early?

To ensure development of normal body image.

Parallel play

Toddlers play alongside, not with, other children.

1. Parents are often confused by the terms growth and development and use the terms interchangeably. Based on the nurse's knowledge of growth and development, the most appropriate explanation of development is • a child grows taller all through early childhood. • a child learns to throw a ball overhand. • a child's weight triples during the first year. • a child's brain increases in size until school age.

a child learns to throw a ball overhand

The most consistent indicator of pain in infants is increased respirations. increased heart rate. clenching the teeth and lips. a facial expression of discomfort.

a facial expression of discomfort. A facial expression of discomfort is the most consistent behavioral manifestation of pain in infants. The respiratory pattern may be markedly variable in an infant in pain and, therefore, is not a consistent indicator of pain. The heart rate may initially decrease in some infants with pain and then increase; therefore, it is not a consistent indicator of pain. Clenching the teeth and lips is a sign of pain often assessed in toddlers, not infants.

The most appropriate method for a nurse to use to view the tonsils and oropharynx of a 6-year-old child is to ask child to open mouth wide and say "Ahh." ask child to open mouth wide, and then place tongue blade in the center back area of the tongue. examine mouth when child is crying to avoid use of tongue blade. pinch nostrils closed until child opens mouth, then insert tongue blade.

ask child to open mouth wide and say "Ahh." If the child is cooperative, the child can open the mouth and move the tongue around for the examiner. No tongue blade is necessary to visualize the tonsils and oropharynx if the child cooperates. During crying, there is insufficient opportunity to completely visualize the tonsils and oropharynx. It is traumatic to pinch the nostrils closed until the child opens the mouth. There is no reason to use such measures, especially with cooperative children.

12. In terms of genetic presentations, if a disease pattern exists without known correlation of symptoms, this would be characterized as a • syndrome. • association. • sequence. • mutation.

association.

The nurse is assessing a 3-year-old African-American child who is being seen in the clinic for the first time. The child's height and weight are in the 20th percentile on the commonly used growth chart from the National Center for Health Statistics. When interpreting the data, the nurse recognizes child's growth is within normal limits. child's growth is not within normal limits. growth chart is not accurate for African-American children. growth chart is not useful until several measurements are plotted over time.

child's growth is within normal limits. The growth charts are population based and include all children without regard to race or ethnicity. A child's growth within the 20th percentile is within the normal range. Children from different ethnic and racial groups are included, making the growth chart representative for all groups. The growth chart is useful both for screening and for assessment over time.

A nurse is starting an intravenous (IV) line for a school-age child with cancer. The child says, "I have had a million IVs. They hurt." The nurse's response should be based on the knowledge that children tolerate pain better than adults. children become accustomed to painful procedures. children often lie about experiencing pain. children often demonstrate increased behavioral signs of discomfort with repeated painful procedures.

children often demonstrate increased behavioral signs of discomfort with repeated painful procedures. Children with chronic illnesses are more likely to identify invasive procedures as stressful compared with children with acute illnesses. There are no data to support the theory that children tolerate pain better than adults. The child has increasing difficulty with numerous and repeated painful procedures rather than becoming accustomed to them. Pain is whatever the experiencing person defines it to be.

The primary reason for universal screening of young children for lead poisoning is that children with lead poisoning rarely have symptoms. water and food in the United States are usually contaminated with lead. most children are exposed to lead through herbal products. most children in the United States are exposed to toxic amounts of lead.

children with lead poisoning rarely have symptoms. In the early stages of lead poisoning, children are asymptomatic. Water and food in the United States are not highly contaminated. Risks are homes painted before 1978, when painting products still had high lead levels. Universal screening will identify children who may receive lead via herbal supplements, if applicable. Universal screening will identify children who may receive lead via environmental exposure, if applicable.

7. During their school-age years, children best understand concepts that can be seen or illustrated. The nurse knows this type of thinking is termed as • concrete operations. • preoperational. • school-age rhetoric. • formal operations.

concrete operations

The appropriate direction to pull the pinna of an infant during an otoscopic examination is down and back. down and forward. up and forward. up and back.

down and back. The correct position for an infant's ear examination is to pull the pinna down and back. Pulling the pinna down and forward is the correct position for a child age 3 years and over. Pulling the pinna up and forward will not allow sufficient visualization of the ear. Pulling the pinna up and back will not allow sufficient visualization of the ear.

A 2-year-old child has recently started having temper tantrums where breath holding occurs and occasionally fainting. The most appropriate action by the nurse is to refer the child for a respiratory evaluation. refer the child for a psychological evaluation. explain to the parent that this is not harmful. explain to the parent that the child is spoiled.

explain to the parent that this is not harmful. The rising carbon dioxide levels restart the breathing process when a child holds his or her breath; therefore, the process is self-limiting and not harmful. A respiratory evaluation is not indicated for this toddler. Temper tantrums are part of this developmental stage as the toddler asserts his or her independence. A psychological evaluation is not warranted. Temper tantrums are part of this developmental stage as the toddler asserts independence. There are no data to indicate that the child is spoiled.

Guidelines for a nurse using an interpreter in developing a care plan for an 8-year-old admitted to rule out epilepsy include explaining to the interpreter what information is necessary to obtain from the patient and family. encouraging the interpreter to ask several questions at a time to make the best use of time. not giving the interpreter too much information so that the interview evolves. discouraging the interpreter and client from discussing topics that are deemed irrelevant to the original intent of the interview.

explaining to the interpreter what information is necessary to obtain from the patient and family. The interpreter should be given guidance as to what information is necessary to obtain during the interview. One question should be asked at a time, leaving sufficient time for the family to answer. The interpreter should not have to guess what to ask and what information to obtain during the interview. The interpreter should gain as much information from the family as they are willing to share based on the questions posed. Limits should not be placed on the interview.

A child who has been receiving morphine by the intravenous (IV) route will now start receiving it orally. In order for equianalgesia (equal analgesic effect) to be achieved, the oral dose will be same as the IV dose. greater than the IV dose. one half of the IV dose. one fourth of the IV dose.

greater than the IV dose. When the route of morphine administration is changed from IV to PO (by mouth), it is essential that the dose be increased to achieve an equianalgesic effect. Oral morphine is not as effective at the same dose as IV morphine. The dosage of morphine is increased, not decreased, when the administration route changes from IV to PO. The dosage of morphine is increased, not decreased, when the administration route changes from IV to PO.

The nurses caring for a child are concerned about the child's frequent requests for pain medication. During a team conference, a nurse suggests that they consider administering a placebo instead of the usual pain medication. This decision should be based on knowledge that it is unjustified and unethical to administer placebos instead of pain medication. the absence of a response to a placebo means the child's pain has an organic basis. a positive response to a placebo will not occur if the child's pain has an organic basis. administering a placebo instead of the usual pain medication is effective in determining whether a child's pain is real.

it is unjustified and unethical to administer placebos instead of pain medication. Placebos should never be given by any route in the assessment or management of pain. The response to a placebo is not a measure of the origin of pain and should never be used as a means of assessing pain. The response to a placebo is not a measure of the origin of pain and should never be used as a means of assessing pain. Placebos should never be given as a means to determine whether pain is real. Individuals respond differently to placebos; therefore, the patient's response may not be an accurate measure of pain.

A child who is terminally ill with bone cancer is in severe pain. Nursing interventions should be based on the knowledge that children tend to be overmedicated for pain. giving large doses of opioids causes euthanasia. narcotic addiction is common in terminally ill children. large doses of opioids are justified when there are no other treatment options.

large doses of opioids are justified when there are no other treatment options. Large doses of opioids may be needed because the child has become physiologically tolerant to the drug, requiring higher doses to achieve the same degree of pain control. Pain is considered the fifth vital sign, and management of pain is critical to treatment of a child with bone cancer. Continuing studies report that children are consistently undermedicated for pain. The dosage of opioids is titrated to relieve pain, not cause death. Addiction refers to a psychological dependence on the narcotic medication, which does not occur in terminal care.

The nurse needs to take the blood pressure of a small child. Of the cuffs available, one is too large and one is too small. The best nursing action is to use the small cuff. use the large cuff. use either cuff, using palpation method. locate the proper-sized cuff before taking the blood pressure.

locate the proper-sized cuff before taking the blood pressure. To obtain an accurate blood pressure reading, it is preferable to use the proper-sized cuff. Therefore, locating one before taking the blood pressure is the best nursing action. The smaller cuff gives a falsely increased blood pressure and is not the method of choice. The larger cuff, which may give a falsely lowered blood pressure, is preferable to the smaller cuff, which gives a falsely increased blood pressure, but neither is the method of choice. Auscultation is preferred to palpation.

A parent is concerned because her 18-month-old daughter who was previously a "good eater" by her accounts is now being very picky during meal times. Meal time patterns of intake vary from one day to the next with eating large amounts then hardly eating at all. Based on this information, the nurse would suspect that the toddler probably has an ear infection so the parent should not be concerned. refer the parent to the pediatrician for a diagnostic work up. tell the mother that this type of behavior is associated with regression. may be exhibiting physiological anorexia which is a common finding during this time period.

may be exhibiting physiological anorexia which is a common finding during this time period. These findings are indicative of physiological anorexia as during this time period, many toddlers demonstrate variation in food intake, strong food preferences and picky eating behaviors. There is no correlation with ear infections causing this type of eating behavior. A diagnostic work up is not warranted and this behavior does not demonstrate regression.

Nonpharmacologic strategies for pain management may reduce pain perception. make pharmacologic strategies unnecessary. usually take too long to implement. trick children into believing they do not have pain.

may reduce pain perception. Nonpharmacologic techniques for pain management may help the child with associated fears and stress related to pain. The strategies may provide assistance with coping that may reduce the perception of pain, decrease anxiety, and increase the effectiveness of medications. The child with moderate or severe pain will require pharmacologic intervention. The child should be taught nonpharmacologic pain management strategies before pain occurs, therefore reducing the implementation time. The child will still have the pain, but the perception may be altered.

When assessing a preschooler's chest, the nurse would expect respiratory movements to be chiefly thoracic. anteroposterior diameter to be equal to the transverse diameter. intercostal retractions on respiratory movement. movement of the chest wall to be symmetric bilaterally and coordinated with breathing.

movement of the chest wall to be symmetric bilaterally and coordinated with breathing. The preschool-aged child should have symmetric chest movement bilaterally and a coordinated breathing pattern. At this age, breathing is a coordinated function and is primarily abdominal or diaphragmatic. Thoracic breathing occurs in older children, particularly girls. The anteroposterior diameter is equal to the transverse diameter in infants. As the child grows, the chest normally increases in the transverse direction; therefore, the anteroposterior diameter is less than the lateral diameter. Intercostal retractions are indicative of respiratory distress.

When explaining the proper restraint of toddlers in motor vehicles to a group of parents, the nurse should include wearing safety belts snugly over the toddler's abdomen. placing the car seat in the front passenger seat if there is an air bag. using lap and shoulder belts when child is over 3 years of age. placing the car seat in the back seat of the car facing forward.

placing the car seat in the back seat of the car facing forward. Car seats are required for toddlers to prevent injury in case of a motor vehicle accident. The car seat should be placed in the back seat, forward facing. Safety belts can cause injuries if they are placed over a toddler's abdomen. Car seats should be in rear of the car because air bags can injure the toddler. Three-year-olds should be restrained in car seats.

A toddler is exploring the environment but returns to his mother within a few minutes of exploration. This finding would be noted as an example of separation anxiety. separation. rapprochement. individuation.

rapprochement. Rapprochement occurs when the child returns to the mother for reassurance following exploration of the environment. Separation anxiety is when the child experiences anxiety based on separation from the parent or significant figure. Separation refers to the emergence of the child as a separate figure from the mother or parent. Individuation refers to the emergence of the child by expressing their own individual characteristic.

The nurse is caring for a comatose child with multiple injuries. The nurse should recognize that pain cannot occur if child is comatose. may occur if child regains consciousness. requires astute nursing assessment and management. is best assessed by family members who are familiar with child.

requires astute nursing assessment and management. Because the child cannot communicate pain through one of the standard pain rating scales, the nurse must focus on physiologic and behavioral manifestations to accurately assess pain. Pain can occur in the comatose child. The child can be in pain while comatose. The family can provide insight into the child's different responses, but the nurse should be monitoring physiologic and behavioral manifestations.

A nurse is conducting a health history on an adolescent. Components of the health history include Select all that apply. sexual history. review of systems. physical assessment. growth measurements. family medical history.

sexual history. review of systems. growth measurements. family medical history. Sexual history is a component of the health history. Review of systems is a component of the health history. Review of family medical history is a component of the health history. Physical assessment is a component of the physical examination. Growth measurements are a component of the physical examination.

The nurse should teach parents of toddlers how to prevent poisoning by instructing them to consistently use safety caps. store poisonous substances in a locked cabinet. keep ipecac in the home. store poisonous substances out of reach.

store poisonous substances in a locked cabinet. This is an appropriate action to prevent the curious toddler from getting into poisonous substances and products. Not all poisonous substances have safety caps, and safety caps are not always foolproof. Ipecac does not prevent poisoning and is not recommended as a treatment for poisoning. Toddlers can climb and are curious; therefore, storing substances out of reach only does not eliminate the potential for poisoning.

15. According to Erikson, infancy is concerned with acquiring a sense of • trust. • industry. • initiative. • separation.

trust

10. The nurse is observing a child who appears to be daydreaming while seated in a chair in the clinic waiting for her scheduled appointment with her mother. This behavior is noted as being an example of • pretend play. • dramatic play. • unoccupied behavior. • skill play.

unoccupied behavior.

For a toddler with sleep problems, the nurse should suggest that the parents use a transitional object at bedtime. vary the bedtime ritual. restrict stimulating activities throughout the day. explain away their fears.

use a transitional object at bedtime. Transitional objects may help ease the toddler's anxiety and facilitate sleep. A consistent set of bedtime rituals will facilitate a toddler's sleep. Toddlers should have stimulating physical activity during the daytime in order for them to sleep at night. Toddlers do not understand verbal explanations, so parents cannot explain away their fears.

Growth and Development

● 4 months ○ Gross motor- almost no head lag, balances head well while sitting, rolls from back to side, able to sit erect if propped up ○ Fine motor- inspects and plays with hands; pulls clothing or blanket over face in play, Tries to reach objects with hand but overshoots, grasps object with both hands, plays with rattle placed in hand and shakes it but cannot pick it up if dropped, can carry objects to mouth ● 7 months ○ Gross motor- When supine, spontaneously lifts head off surface, sits leaning forward on both hands, When prone, bears weight on one hand, Sits erect momentarily bears weight on feet, bounces actively when held in standing position ○ Fine motor- transfers object from one hand to another, Has unidextrous approach and grasp, Holds two cubes more than momentarily, bangs cubes on table, rakes at a small object ● 8 months ○ Gross motor- Sits steadily unsupported. Readily bears weight on legs when supported; may stand holding on to furniture. Adjusts posture to reach an object ○ Fine motor- Has beginning pincer grasp using index, fourth, and fifth fingers against lower part of thumb. Releases objects at will. Rings bell purposely. Retains two cubes while regarding third cube. Secures an object by pulling on a string. Reaches persistently for toys out of reach ● 12 months ○ Gross motor- walks with one hand held, stands alone momentarily, may attempt first step alone. Can go from standing to sitting without help ○ Fine motor- Releases cubes in cup. Attempts to build two block tower but fails. Can turn pages in a book- many at a time.

Addison's Disease

● Adrenal insufficiency is usually caused by a genetic disorder causing one of the following: abnormal development of the adrenal gland, dysfunctional synthesis of adrenal hormones, resistance to ACTH, or damage to the adrenal cortex. Other causes include infection, a destructive lesion of the adrenal gland, or an autoimmune process. Because 90% of adrenal tissue must be nonfunctional before signs of insufficiency are manifested, onset of symptoms is often gradual. ● Treatment- replacement of glucocorticoids (cortisol) and mineralocorticoids (aldosterone) ● Watch for cushing syndrome for excess cortisol

Parenting Styles

● Authoritarian ○ Parents try to control their children's behavior and attitudes through unquestioned mandates ○ This style may be associated with defiant and antisocial behavior ● Permissive ○ Parents exert little or no control over their children's actions ○ They rarely punish the children ● Authoritative ○ Parents combine practices from both of the other styles ○ They respect the individuality of each child and allow the child to voice objections to family standards or regulations ○ Parental controls is firm and consistent ○ Control is focused on the issue not on withdrawal of love or the fear of punishment

Iron Deficiency Anemia

● Can be caused by any number of factors that decrease the supply of iron, impair its absorption, increase the body's need for iron, or affect the synthesis of Hgb ● Iron supplementation to exclusively breastfed infants by 4 months of age because breast milk is a low iron source ● Iron fortified formula ● Best first solid food source of iron is commercial iron fortified cereal

Myelomeningocele

● Develops during the first 28 days of pregnancy when the neural tube fails to close and fuse at some point along its length ● Severe form of spina bifida in which the spinal cord and nerves develop outside of the body and are contained in a fluid filled sac that is visible outside of the back area

Procedures for Cardiac Diagnosis

● Echocardiography ○ Most common ○ Ultra high frequency sound waves to produce an image of the hearts structure ● Electrocardiography ○ Measure the electrical activity of the heart; provides a graphic display; and supplies information on heart rate and rhythm, abnormal rhythms or conduction, ischemic changes and other information. ● Cardiac catheterization ○ Invasive procedure in which a radiopaque catheter is introduced through a large bore needle into a peripheral vessel and then guided into the heart with the aid of a fluoroscopy. ○ Postprocedural care: pulses, temp., color of extremities, vital signs, BP, dressing, fluid intake, blood glucose levels. Must remain straight for 4 to 6 hours following the procedure ○ Lay pt flat and apply pressure. Hgb and Hct should be checked.

Defects of the Genitourinary Tract

● Hypospadias ○ Urethral opening located behind glans penis or anywhere along ventral surface of penile shaft ● Phimosis ○ Narrowing or stenosis of the preputial opening of the foreskin that prevents retraction of the foreskin over the glans penis. ● Hydrocele ○ Presence of peritoneal fluid in the scrotum between the parietal and viceral layers of the tunica vaginalis and is the most common cause of painless scrotal swelling in children

Types of Dehydration

● Isotonic ○ Occurs in conditions in which electrolyte and water deficits are present in approximately balanced proportions ○ Primary form of dehydration in children ● Hypotonic ○ Occurs when the electrolyte deficit exceeds the water deficit ● Hypertonic ○ Results from water loss in excess of electrolyte loss and is usually caused by proportionately larger loss of water or a larger intake electrolytes ○ Most dangerous and requires much more specific fluid therapy

Piaget's Cognitive Development

● Sensorimotor ○ Intellectual development; they develop a sense of cause and effect as they direct behavior towards objects ○ Problem solving is primarily "trial and error" ● Preoperational ○ Egocentrism and it is the inability to put oneself in the place of another ○ They are unable to see things from any perspective other than their own

Genetics

● Syndrome ○ A recognized pattern of anomalies resulting from a single specific cause (Down syndrome, fetal alcohol syndrome) ● Association ○ A nonrandom pattern of malformations for which a cause has not been determined (cardiac defects, fistula, renal and limb defects, etc.) ● Sequence ○ When a single anomaly leads to a cascade of additional anomalies, the pattern of defects is referred to as a sequence

Sickle-Cell Anemia

● The clinical manifestations are obstruction caused by the sickle RBCs with other cells, vascular inflammation, and increased RBC destruction ● Causes vaso occlusion ● Most acute symptoms of the disease occur during periods of exacerbation called crises ○ vaso occlusive , acute splenic sequestration, aplastic, hyperhemolytic, cerebrovascular accident, acute chest syndrome, and infection ● Vaso Occlusive crisis is a painful episode, characterized by ischemia causing mild to severe pain that may last from minutes to days or longer ○ Swelling in hands and feet is often the first painful manifestation

Diarrhea

● The major goals in the management of acute diarrhea include assessment of fluid and electrolyte imbalance, rehydration, maintenance fluid therapy, and reintroduction of an adequate diet. ● Treat infants and children with acute diarrhea and dehydration first with oral rehydration therapy (ORT)

Diabetes Insipidus

● Under secretion of antidiuretic hormone or vasopressin producing a state of uncontrolled diuresis and excessive thirst and water intake ● Cardinal signs of DI are polyuria and polydipsia; early sign would be enuresis ● In infants the initial symptom is irritability that is relieved with feedings of water but not milk ● Diagnostic test- water deprivation test; if it is positive give a test dose of desmopressin ● Treatment- hormone replacement using desmopressin (PO, intranasally, or parentally) ● Nursing care management- frequent measures of weight, electrolytes, BUN, hematocrit, and urine specific gravity; monitor I&O

Physical Examination: Age-specific approaches during childhood (Infant & Toddler)

●Infant -Position: Before able to sit alone: supine or prone, preferably in parent's lap; before 4-6 months, can place on examining table. After able to sit alone: sitting in parent's lap whenever possible; or on table with parent in full view -Sequence: If quiet, auscultate heart, lungs, and abdomen. Record heart and respiratory rates. Palpate and percuss same areas. Proceed in usual head-to-toe direction. Perform traumatic procedures last (eyes, ears, mouth). Elicit reflexes as body part is examined. Elicit Moro reflex last. -Preparation: Completely undress if room temperature permits. Leave diaper on male infant. Gain cooperation with distraction, bright objects, rattles, talking. Smile at infant; use soft, gentle voice. Use pacifier (if used) or bottle with feeding (if bottle feeding). Enlist parent's aid for restraining to examine ears, mouth. Avoid abrupt, jerky movements. ●Toddler -Position: Sitting on parent's lap or standing by parent. Prone or supine in parent's lap -Sequence: Inspect body area through play: "Count fingers," "tickle toes." Use minimum physical contact initially. Introduce equipment slowly. Auscultate, percuss, palpate whenever quiet. Perform traumatic procedures last. -Preparation: Have parent remove outer clothing. Remove underwear as body part is examined. Allow toddler to inspect equipment; demonstrating use of equipment is usually ineffective. If uncooperative, perform procedures quickly. Use restraint when appropriate; request parent's assistance. Talk about examination if cooperative; use short phrases. Praise for cooperative behavior.

Physical Examination: Age-specific approaches during childhood (Preschool, School-age, and Adolescent)

●Preschool Child -Position: Prefer standing or sitting. Usually cooperative prone or supine. Prefer parent's closeness. -Sequence: If cooperative, proceed in head-to-toe direction. If uncooperative, proceed as with toddler. -Preparation: Request self-undressing. Allow to wear underwear if shy. Offer equipment for inspection; briefly demonstrate use. Make up story about procedure ("I'm seeing how strong your muscles are" [blood pressure]). Use paper-doll technique. Give choices when possible. Expect cooperation; use positive statements ("Open your mouth."). ●School-age Child -Position: Prefer sitting. Cooperative in most positions. Younger child prefers parent's presence. Older child may prefer privacy -Sequence: Proceed in head-to-toe direction. May examine genitalia last in older child. -Preparation: Respect need for privacy. Request self-undressing. Allow to wear underwear. Give gown to wear. Explain purpose of equipment and significance of procedure, such as otoscope to see tympanic membrane, which is necessary for hearing. Teach about body function and care. ●Adolescent -Position: Same as for school-age child. Offer option of parent's presence -Sequence: Head-to-toe direction. May examine genitalia last. -Preparation: Allow to undress in private. Give gown. Expose only area to be examined. Respect need for privacy. Explain findings during examination ("Your muscles are firm and strong."). Matter-of-factly comment about sexual development ("Your breasts are developing as they should be."). Emphasize normalcy of development. Examine genitalia as any other body part at end.

Austims Spectrum Disorders (ASD)

-ASDs are complex neurodevelopmental disorders of unknown etiology. The DSM-5 revised the definition of ASD based on two behavior domains: difficulties in social communication and social interaction and unusually restricted, repetitive behavior, interest, or activities. -ASD is frequently diagnosed in toddlers because their atypical development is being recognized early -Clinical Manifestations: Children with ASD demonstrate core deficits primarily in social interactions, communication, and behavior. Failure of social interaction and communication development is one of the hallmarks of ASD. Parents of autistic children have reported that their child showed less interest in social interaction (e.g., abnormal eye contact, decreased response to own name, decreased imitation, usual repetitive behavior) and had verbal and motor delay. Children with ASD may have significant gastrointestinal symptoms. Constipation is a common symptom and can be associated with acquired megarectum in children with ASD. Children with autism do not always have the same manifestations, from mild forms requiring minimal supervision to severe forms in which self-abusive behavior is common. The majority of children with autism have some degree of CI, with scores typically in the moderate to severe range. Despite their relatively moderate to severe disability, some children with autism (known as savants) excel in particular areas, such as art, music, memory, mathematics, or perceptual skills, such as puzzle building. -Communication impairments are a common sign in children with ASD that may range from absent to delayed speech. Any child who does not display language skills such as babbling or gesturing by 12 months old, single words by 16 months old, and two-word phrases by 24 months old is recommended for immediate hearing and language evaluation.

Dysmenorrhea

-Dysmenorrhea, pain during or shortly before menstruation, is one of the most common gynecologic problems in women of all ages. -Dysmenorrhea is associated with menarche prior to 12 years of age, nulliparity, heavy menses, pelvic inflammatory disease (PID), BMI greater than 20, smoking, and depression. -Dysmenorrhea is differentiated as primary or secondary: Primary: a condition associated with ovulatory cycles and has a biochemical basis and arises from the release of prostaglandins with menses. The pain begins with the onset of menstruation and lasts 8 to 48 hours. Primary dysmenorrhea usually appears 6 to 12 months after menarche when ovulation is established. Secondary: defined as painful menses associated with a pathologic condition, such as adenomyosis, endometriosis, PID, endometrial polyps, or fibroids.

Fragile X Syndrome

-FXS is the most common inherited cause of cognitive impairment and the second most common genetic cause of CI or intellectual disability after Down syndrome. -The inheritance pattern has been termed X-linked dominant with reduced penetrance. -Clincal Manifestations: Physical Features: Increased head circumference. Long, wide, or protruding ears. Long, narrow face with prominent jaw. Strabismus. Mitral valve prolapse, aortic root dilation. Hypotonia. In postpubertal males, enlarged testicles Behavioral Features: Mild to severe cognitive impairment (CI). Speech delay; may be rapid speech with stuttering and word repetition. Short attention span, hyperactivity. Hypersensitivity to taste, sounds, and touch. Intolerance to change in routine. Autistic-like behaviors, such as social anxiety and gaze aversion. Possible aggressive behavior.

Tracheostomy Care

-Hyperoxygenate first -The airway must remain patent and may require frequent suctioning to remove mucus plugs and excessive secretions. -Don't suction longer than 5 seconds.

Colic

-The condition is generally described as abdominal pain or cramping that is manifested by loud crying and drawing the legs up to the abdomen. -Parents typically express dissatisfaction with the amount of time the infant spends crying each day. -Colic can be characterized by crying: more than 3 hours per day, for more than 3 days per week, and for more than 3 weeks. -Symptoms may increase in the late afternoon or evening; however, in some infants the onset of symptoms occurs at varying times. -Colic is more common in infants younger than 3 months old and in infants with difficult temperaments.

Phenylketonuria (PKU) testing is most accurate when obtained A. 20 hours after birth. B. 24 to 48 hours after birth. C. during newborn transition. D. at the newborns two-week checkup.

. 24 to 48 hours after birth.

Parents have brought their 6 months old daughter in for a 1 week follow up for treatment of diaper dermatitis. The parents state that they have followed all directions but that it just doesn't seem to be getting any better. The nurse examines the infant and finds no resolve of the condition. Based on this finding, the nurse suspects that A) the infant may also have a Candida infection. B) additional teaching may be needed for the parents in order to validate that they followed through with instructions. C) suggest to the parents that a heat lamp maybe needed to resolve the problem. D) have the parents continue their treatment as it may take more than 1 week to resolve.

A

The most appropriate comfort intervention for a child with severe itching related to chickenpox is A) give an antipruritic medication such as diphenhydramine (Benadryl). B) apply thick coat of pramoxine (Caladryl) lotion over open lesions. C) give aspirin or acetaminophen (Tylenol). D) encourage frequent warm baths.

A

The most important prevention method for the spread of any communicable disease is A) hand washing. B) isolation from infectious agents. C) immunizations as secondary prevention. D) use of appropriate broad spectrum antibiotics.

A

The school nurse is concerned about an outbreak of chickenpox because two children at the school have cancer and are immunodeficient from chemotherapy. The most appropriate recommendation by the school nurse is that A) varicella-zoster immune globulin (VZIG) to prevent chickenpox. B) acyclovir (Zovirax) should be taken to minimize the symptoms of chickenpox. C) temporarily stopping chemotherapy will allow the immune system to recover. D) no precautions necessary.

A

Which clinical finding would alert the nurse to the possibility that the pediatric patient has conjunctivitis as a result of a foreign object body? A)Tearing of the right eye B) Bilateral swollen lids C) Inflamed conjunctiva of both eyes D) Crusting of eyelids

A

Which food combination will generally provide the appropriate amounts of essential amino acids for an individual who is a vegetarian? A) Grains and legumes B) Grains and vegetables C) Legumes and vegetables D) Milk products and fruit

A

Which is the most descriptive of kwashiorkor? A) Kwashiorkor has a multifactorial etiology. B) Kwashiorkor occurs primarily in breastfed infants. C) Kwashiorkor results from excessive amounts of vitamin K. D) Kwashiorkor is related to inadequate calories, not adequate protein.

A

Parents are concerned about the behavior of their preschool child as he is exhibiting aggressive behavior in interactions with other children. Which information would be relevant in determining if the behavior represents an abnormal pattern? (SATA) A: How many times this type of behavior has occurred in recent weeks? B: Does the behavior seem to be escalating? C: How long has this behavior been going on? D: Do you think that the behavior is warranted considering the situation that has occurred? E. Does the behavior interfere with social functioning?

A, B, C, E In terms of making an evaluation as to whether or not this aggressive behavior is a problem, the nurse would want to obtain information related to quantity, severity, distribution, onset and duration. Asking the parent if they think the behavior is justified given the situation does not address any of the aforementioned variables of concern but rather reflects a judgment value.

Phototherapy is indicated for the clinical treatment of A. ABO incompatibilities so as to reverse the hemolytic process. B. cyanosis due to congenital cardiac disease presentations. C. coagulation disorders as a result of clotting factor deficiencies. D. restoration of fluid balance due to jaundice.

A. ABO incompatibilities so as to reverse the hemolytic process.

A nurse is reviewing a family history and notes that the parents are married and living in the same household. This description would be documented as which type of family relationship? A. Affinal B. Family of origin C. Communal D. Consanguineous

A. Affinal

Which has had the greatest impact on reducing infant mortality in the United States? A. Improvements in perinatal care B. Decreased incidence of congenital abnormalities C. Better maternal nutrition D. Improved funding for health care

A. Improvements in perinatal care

A newborn has antibiotics ordered for possible sepsis. Which nursing intervention would the nurse prepare to implement? Select all that apply. A. Wait for blood culture results before starting antibiotics. B. Restrict parental visits C. Monitor temperature D. Monitor oxygen levels E. Monitor blood sugars for signs of hypoglycemia

A. Wait for blood culture results before starting antibiotics. C. Monitor temperature D. Monitor oxygen levels E. Monitor blood sugars for signs of hypoglycemia

Which is an appropriate nursing intervention when caring for an infant with neonatal abstinence syndrome (NAS)? A. Wrap the infant snugly. B. Initiate an early stimulation program. C. Place the infant in an infant seat for feedings. D. Teach the mother how to provide tactile stimulation.

A. Wrap the infant snugly.

The nurse receives a report about a newborn stating that the mother has type O+ blood and the infant has type B+ blood. The infant is at highest risk for developing? A. hyperbilirubinemia. B. hypoglycemia. C. erythema toxicum. D. phenylketonuria.

A. hyperbilirubinemia.

When preparing to administer Hepatitis B vaccine to a newborn, the nurse should? Select all that apply. A. initiate an immunization record. B. confirm the hepatitis B status of the newborn's mother. C. obtain a syringe with a 25-gauge, 5/8-inch needle. D. assess the dorsogluteal muscle as the preferred site for injection. E. confirm that the newborn's mother has signed the informed consent.

A. initiate an immunization record. B. confirm the hepatitis B status of the newborn's mother. C. obtain a syringe with a 25-gauge, 5/8-inch needle. E. confirm that the newborn's mother has signed the informed consent.

Which is descriptive of the nutritional requirements of preschool children? A: The quality of the food consumed is more important than the quantity. B: Nutritional requirements for preschoolers are different from requirements for toddlers. C: The requirement for calories per unit of body weight increases slightly during the preschool period. D: The average daily intake of preschoolers should be about 3000 calories.

A: The quality of the food consumed is more important than the quantity. It is essential that the child eat a balanced diet with essential nutrients; the amount of food is less important than the quality of the food. Requirements are similar for both toddlers and preschoolers. The caloric requirement decreases slightly for preschoolers. The average intake is about 1800 calories each day for preschoolers.

The primary goals in the nutritional management of children with failure to thrive (FTT) are (Select all that apply.) A) allow for catch-up growth. B) correct nutritional deficiencies. C) achieve ideal weight for height. D) restore optimum body composition. E) educate the parents or primary caregivers on child's nutritional requirements.

ABCDE

Which herbal therapies are associated with increased milk production? (Select all that apply.) A) Blessed thistle B) Fennel C) St. John's Wort D) Fenugreek E) Echinacea

ABD

ABO incompatibility

ABO incompatibility ● The incompatibility is between a mother with O blood group and an infant with A or B blood group ● Anemia results from the hemolysis of large number of erythrocytes, and hyperbilirubinemia and jaundice result from the liver's inability to conjugate and excrete the excess bilirubin Hyperbilirubinemia ● Refers to an excessive level of accumulated bilirubin in the blood and is characterized by jaundice ● Phototherapy ○ Light promotes bilirubin excretion

In order for an infant/child to formulate an attachment with another human being, they must (Select all that apply.) A) discriminate self between individuals. B) understand moral principles of right versus wrong. C) achieve object permanence. D) understand principles of time. E) recognize themselves in the mirror.

AC

Infants in the first few days of life are expected to have a weight loss between 5 and 10% based on the following principles (Select all that apply.) A) increased renal tubular function. B) enlargement of ECF compartment. C) increased glomerular filtration rate. D) shivering thermogenesis. E) decrease in percentage of body water.

ACE

Why are blood pressure readings taken frequently in a child with acute glomerulonephritis?

Acute hypertension must be anticipated and identified.

When is the time frame for starting primary immunizations for a baby who was just born? A) Following birth or up to two weeks of age B) At four weeks C) At two months D) Recommendation based on gender and weight protocols

A

17. The parent of a 12-month-old says to the nurse, "He pushes the teaspoon right out of my hand when I feed him. I can't let him feed himself, he makes too much mess." The most appropriate response by the nurse is • "It's important not to give in to this kind of temper tantrum at this age." • "Maybe you need to try a different type of spoon, one designed for children." • "It's important to let him make a mess. Just don't worry about it so much." • "He is at the age when he should begin to feed himself. Let's think of ways to make the mess more tolerable."

"He is at the age when he should begin to feed himself. Let's think of ways to make the mess more tolerable."

What is an appropriate nursing intervention for a child with nephrotic syndrome on bed rest?

Adjust activities to the child's tolerance level.

A parent is concerned that her toddler may become obese if high fat or sugary foods are included in the diet. Which food selection should still be included in a child's dietary plan during this stage of life? A) Sweetened fruit juices B) Whole milk C) Low fat milk D) Processed cakes and muffins

B

A group of nurses are reviewing potential health environmental for the spread of noscomial infections in the hosptial. Which equipment should the nurse include as having a high risk potential for transmission? (Select all that apply.) A) Oral temperature probe B) Disposable tongue blade C) Nurse's station desktop D) Stethoscope E) Computer keyboard

C, D, E

The nurse is interviewing the mother of Adam, age 9 years. Which question would be the most appropriate as the nurse begins to assess Adam's school performance? "Did Adam go to preschool?" "Does Adam have problems at school?" "How is Adam doing in school?" "How well does Adam seem to be doing in school?"

"How is Adam doing in school?" How is Adam doing in school? is an open-ended question without any descriptive terms that may limit the mother's responses. Did Adam go to preschool? is a close-ended question, which will elicit a yes or no answer. Does Adam have problems at school? is a close-ended question that implies Adam is not doing well. How well does Adam seem to be doing in school? is a close-ended question that will have a short answer and assumes Adam is doing well.

The nurse is interviewing the parents of a toddler and wants to determine the child's feeding preferences during meal time. Which statement made by the nurse is an example of directed focus? "I know we have discussed your son's eating habits but can we now discuss what Sam like to eat for lunch?" "How much time does it take for Sam to finish his meals?" "Would Sam prefer hot dogs or chicken nuggets, if given a choice?" "Would Sam prefer pudding as opposed to cake?"

"I know we have discussed your son's eating habits but can we now discuss what Sam like to eat for lunch?" Asking an open ended question following a statement help to direct the focus and obtain more information. The other provided options are examples of close-ended questions and as such would lead to a limited information related to specific findings of time and individual food preferences.

The nurse is providing education to a parent of a 10-month-old infant with the diagnosis of cow's milk allergy. What will be included in the teaching? (Select all that apply.) A) Reading of all food labels to avoid products with milk. B) Use of milk to desensitize the child. C) Introduction of soy-based products to replace milk. D) Signs and symptoms associated with potential accidental ingestion of milk.

A

A woman has given birth to a healthy boy. When the nurse brings the newborn to its mother for feeding, she is shocked at the elongated appearance of the baby's head. The nurse should reassure her by saying which of the following? "All newborn babies' heads are shaped this way." "After the soft spot closes, the head will return to normal." "The infant's head is molded during delivery and will return to normal in a few days." "The infant's head shape has changed during delivery, and it will take approximately 6 months for it to return to normal."

"The infant's head is molded during delivery and will return to normal in a few days." Molding of the head takes place as the baby moves through the birth canal, and the head will return to a normocephalic shape within a few days.

A parent brings a 2-year-old to the clinic for a well-child checkup. Which statement by the parent would indicate to the nurse that the parent needs more instruction regarding accident prevention? "We locked all the medicines in the bathroom cabinet." "We turned the thermostat down on our hot water heater." "We placed gates at the top and bottom of the basement steps." "We stopped using the car seat and put our child in the seat belt now that he is older."

"We stopped using the car seat and put our child in the seat belt now that he is older." A car seat should be used until the child weighs 18 kilograms (40 pounds) and is approximately 4 years old. Locking up medicines and any other harmful household products is an appropriate action; therefore, there is no need for further instruction. Turning down the thermostat on the hot water heater is an appropriate action; therefore, there is no need for further instruction. Placing gates at the top and bottom of the basement stairs is an appropriate action; therefore, there is no need for further instruction.

18. Parents of a 4-month-old infant bring the infant to the clinic for a well-baby checkup. Which instruction should the nurse include at this time about injury prevention? • "Never shake baby powder directly on the infant because it can be aspirated into the lungs." • "Do not permit the child to chew paint from window ledges, because the child might absorb too much lead." • "When the child learns to roll over, you must offer supervision whenever the child is on a surface from which the child might fall." • "Keep doors of appliances closed at all times."

"When the child learns to roll over, you must offer supervision whenever the child is on a surface from which the child might fall."

The mother of a child with type 1 diabetes mellitus asks why her child cannot avoid all those "shots" and take pills as an uncle does. The most appropriate response by the nurse is

"Your child needs to have insulin replaced, and the oral hypoglycemics only add to an existing supply of insulin."

Chronic Illness

-Denial as a defense mechanism is a necessary cushion to prevent disintegration and is a normal response to grieving for any type of loss. -Shock and denial can last from days to months, sometimes even longer. -Examples of denial that may be exhibited at the time of diagnosis include: Physician shopping, Attributing the symptoms of the actual illness to a minor condition, Refusing to believe the diagnostic tests, Delaying consent for treatment, Acting happy and optimistic despite the revealed diagnosis, Refusing to tell or talk to anyone about the condition, Insisting that no one is telling the truth, regardless of others' attempts to do so, Denying the reason for admission, Asking no questions about the diagnosis, treatment, or prognosis. -Generally, these mechanisms should be respected as short-term responses that allow individuals to distance themselves from the tremendous emotional impact and to collect and mobilize their energies toward goal-directed, problem-solving behaviors. -Adjustment gradually follows shock and is normally seen with guilt, self-accusation, and anger. -Many parents of children with chronic illnesses experience chronic sorrow, which are feelings of sorrow and loss that recur in waves over time.

A pediatric patient at risk for developing complications of immobility during the postoperative period should have which measures incorporated into the plan of care? (Select all that apply.)

Place in supine position with extremities in alignment, Include range of motion, both active and passive as tolerated, Plan for play activities as tolerated, Maintain hydration during postoperative period.

For which scenario would the expectation of confidentiality by the nurse not be withheld during an interview format? Select all that apply. 15-year-old emancipated minor who wants to discuss birth control methods 14-year-old patient who denies abuse but who presents with multiple bruises over arms and legs which appear to be "defensive type" in nature 16-year-old patient who appears sad and voices despair over having broken up with his boyfriend states he has no options 18-year-old patient who confides in the nurse that she wants to move out and get her own apartment

14-year-old patient who denies abuse but who presents with multiple bruises over arms and legs which appear to be "defensive type" in nature 16-year-old patient who appears sad and voices despair over having broken up with his boyfriend states he has no options Any patient should have the expectation during the interview process that the concept of confidentiality will be provided, regardless of age or gender. However, there are certain exceptions to the concept of confidentiality when there are perceived threats of either violence, self-directed or as a consequence of abuse. Nurses are legally required to report suspected evidence of abuse and/or actions related to suicide. An emancipated minor seeking birth control and an 18 year old who wishes to move out of her home and get an apartment are within the concept of confidentiality.

What is characteristic of fractures in children?

Rapidity of healing is inversely related to the child's age.

In teaching prospective parents about the emergence of gender identity, the nurse should instruct them that this concept emerges at about 12 months of age. 24 months of age. at birth. at 3 years of age.

24 months of age. The concept of gender identity begins at about 2 years of age (24 months), at which children are able to label their own as well as other's genders. It is not inherent at birth or by 1 year of age.

An important nursing intervention in the care of a child with bacterial conjunctivitis is A) intermittent warm, moist compresses to remove crusts on the eye area. B) oral antihistamines to minimize itching. C) application of optic corticosteroids to reduce inflammation. D) continuous warm compresses to relieve discomfort.

A

11. At what age would a child demonstrate the ability to understand the concept of compromise as related to social play interactions? • 2 years of age • 5 to 6 of age • It depends on the child's ability to reason and therefore may vary considerably • It is a learned concept and is typically present by 10 years of age.

5 to 6 of age

A nurse is preparing an educational workshop on atraumatic care in pediatric patient care. The most appropriate nursing intervention to include in the workshop is to A) provide the child play activities for expression of fear and aggression. B) prepare the child that their parents will not be able stay during hospitalization by watching a video. C) help the child to accept the pain associated with any treatments, procedures, or surgery. D) tell the child that the loss of control and privacy in the hospital is temporary.

A

Which method should the student nurse use to apply the principles of cough etiquette in the clinical setting to prevent the potential spread of infection? A) Using tissues when coughing to catch secretions. B) Maintaining a perimeter of 10 feet from patient and visitors when coughing. C) Wearing a surgical mask for all patient contacts even if the student nurse has not overt clinical symptoms of having a cold. D) Covering the nose when coughing.

A

With regard to the nutritional well-being, at what age should the nurse instruct the parents that lifelong learning habits have been fostered? A) By age 3 B) By 1 year of age C) When the child enters kindergarten D) At 6 months of age when solid food introduction takes place

A

Who would be the most appropriate roommate for a 6-year-old child with acute renal failure?

A 5-year-old child who has a fractured femur.

Which finding would be considered to be abnormal with regard to growth and development principles? (Select all that apply.) A) Chest circumference is equal to head circumference at the end of the first year. B) Increased height is most likely to do to size of limbs rather than torso by the end of the first year. C) Increase in weight of the brain about 1.5 times by the end of the first year. D) Growth of heart is doubled by the end of the first year. E) Primitive reflexes remain consistently within the first year of life.

A) Chest circumference is equal to head circumference at the end of the first year. D) Growth of heart is doubled by the end of the first year.

The nursing process is a method of problem identification and problem-solving that describes what the nurse actually does. The six-step nursing process model includes (Select all that apply.) A). planning. B) evaluation. C) diagnosis. D) assessment. E) clinical outcomes identification. F) diagnostic testing.

A, B, C, D, E

A nurse is presenting a class on injury prevention to parents of preschoolers. Which injuries should the nurse identify as occurring in this age group? (SATA) A: Falls B: Drowning C: Poisoning D: Sports injuries E: Tricycle and bicycle accidents

A, B, C, E Falls occur frequently in preschoolers. Closely monitor playground activities such as climbing jungle-gym. Closely supervise around any water and ensure swimming pools are securely fenced to prevent submersion injury. Place all medications and poisons out of reach and in locked cabinets. Administer medications as a drug, not as candy. Keep poison control phone number by telephone. When riding tricycles and bicycles, children often forget not to ride in the streets. Sports injuries occur in older children.

Bright Futures program provides initiatives that focus on health promotion of children by focusing on (Select all that apply.) A) providing a safe environment. B) providing limited mental health services. C) limitations on introduction of health and sexuality teaching so as to assure parental support. D) encouraging use of community resources. E) promotion of healthy eating habits. F) focus on after school activities as a source of engagement.

A, D, E, F

A newborn who is suspected of having atopy would most likely have which diagnostic finding? A) Small for gestational age (SGA) B) Increased levels of IgE in umbilical cord blood C) No family history of allergies D) Precipitous delivery

B

14. Which characteristic best describes the fine motor skills of a 5-month-old infant? • Transfers objects from one hand to another • Crude pincer grasp • Able to build a tower of two cubes • Able to grasp an object voluntarily

Able to grasp an object voluntarily

Which statement is true concerning the increased use of telephone triage by nurses? Telephone triage has led to an increase in health care costs. Emergency department visits are not recommended by nurses, and therefore they are not a component of telephone triage. Access to high-quality health care services has increased through telephone triage. Home care is often recommended when it is not appropriate.

Access to high-quality health care services has increased through telephone triage. The judicious use of telephone triage has decreased the number of unnecessary visits, allowing time for improved care. Health care costs have decreased because of fewer visits to emergency departments. Based on the response to screening questions, the triage nurse determines whether the child needs to be referred to emergency medical services. The nurse can then initiate the call if needed. Home care is recommended only when indicated, based on the screening questions.

A nursing student is discussing the technique of interviewing with his instructor and conveys that he is somewhat reluctant to talk with potential patients as he fears he may have nothing to say and there would be periods of silence. Which statement represents the best response by the nursing instructor in response to the students' expressed concerns? Telling the student that everyone feels like this at first but that the feeling and anxiety will reside during the next interview experience. Encourage the student to practice interviewing technique skills with peers and family members to increase his confidence level. Acknowledge that his reluctance is normal but that the utilization of silence may well eventually represent the ability of a confident interviewer in knowing that sometimes it is equally important to listen rather than to keep talking. Provide the student with practice questions for interviewing and

Acknowledge that his reluctance is normal but that the utilization of silence may well eventually represent the ability of a confident interviewer in knowing that sometimes it is equally important to listen rather than to keep talking. The development of interviewing skills requires acceptance and acknowledgement of one's own strengths and weaknesses. The inclusion of silence is a very important interviewing technique for it allows the interviewer to assess and listen to the patient for subtle information that is driven both by verbal and non-verbal actions. Telling the student that his feelings and anxiety will resolve by the next interview attempt may not be realistic. Practicing interviewing skills while beneficial do not address the concern of silence as a therapeutic option.

Which medication order would require immediate intervention by the nurse in order to prevent potential complications for the pancytopenic cancer pediatric patient? Offer fluids to hydrate the patient Administer Aleve for relief of joint related pain Administer Neupogen as ordered Administer morphine for severe pain as ordered

Administer Aleve for relief of joint related pain NSAIDs are contraindicated for pancytopenic patients as they are thrombocytopenic and are at risk for bleeding. Fluids can be given to help hydrate the patient. Neupogen is a colony stimulating agent which can be given to increase neutrophil activity to help increase white blood cell count. An opioid narcotic, such as morphine can be used for the treatment of severe pain in a pediatric oncology patient.

What is the priority of nursing care for a young child with suspected bacterial meningitis?

Administer antibiotic therapy as soon as it is ordered

What is included in the therapeutic management of a patient with systemic lupus erythematosus (SLE)?

Administration of corticosteroids to control inflammation.

What is an advantage to teach to the family about continuous cycling peritoneal dialysis (CCPD) or continuous ambulatory peritoneal dialysis (CAPD) for adolescents who require dialysis?

Adolescents can carry out procedures themselves.

Which explains the importance of detecting strabismus in young children? Color vision deficit may result. Amblyopia, a type of blindness, may result. Epicanthal folds may develop in the affected eye. Ptosis may develop secondarily.

Amblyopia, a type of blindness, may result. Amblyopia may develop if the eyes do not work together. The brain may ignore the visual cues from one eye, resulting in blindness. Color vision depends on rods and cones in the retina, not muscle coordination. Epicanthal folds are present at birth. Ptosis, or drooping eyelids, is not related to strabismus (or crossed eyes).

What is the priority nursing intervention for an unconscious adolescent with a temperature of 105°F (40.5°C)?

Apply a hypothermia blanket

Which critical element should be included in a plan of care for a pediatric patient who has to be immobilized as part of the course of therapeutic intervention?

Asking the patient to help organize individual care into a schedule.

What is required for the management of pain in a comatose child with multiple injuries?

Astute nursing assessment and management

A nurse is caring for an infant with developmental dysplasia of the hip (DDH). Based on the nurse's knowledge of DDH, which clinical manifestation should the nurse expect to observe? (Select all that apply.)

Asymmetric thigh and gluteal folds, Positive Ortolani and Barlow tests, Shortening of limb on affected side, Negative Babinski sign.

The nurse is ready to begin a physical examination on an 8-month-old infant. The child is sitting contentedly on the mother's lap, chewing on a toy. What should the nurse do first? Elicit reflexes. Auscultate the heart and lungs. Examine the eyes, ears, and mouth. Examine the head, systematically moving toward the feet.

Auscultate the heart and lungs. While the child is quiet, auscultation should be performed. It may disturb or upset the child to elicit reflexes first, making auscultation and the remainder of the physical examination difficult. It may disturb or upset the child to examine the eyes, ears, and mouth first, making auscultation and the remainder of the physical examination difficult. Although most physical examinations proceed from the head to the feet, the nurse should perform the assessment for a child in an order that moves from least disturbing to most disturbing from the child's perspective.

What nursing intervention is used to prevent increased intracranial pressure (ICP) in an unconscious child?

Avoid activities that cause pain or crying

Which term refers to a shared cultural, social, and linguistic heritage? A. Culture B. Ethnicity C. Beliefs D. Socialization

B. Ethnicity

A 4½-year-old boy has been having increasingly frequent angry outbursts in preschool for approximately 8 to 10 weeks. In addition, he is aggressive toward the other children and teachers. His parents ask the nurse for advice. The most appropriate nursing intervention is to: A: explain that this is normal in preschoolers, especially boys. B: refer the child for counseling. C: talk to the preschool teacher to obtain validation for the behavior the parent reports. D: encourage the parent to try more consistent and firm discipline.

B: refer the child for counseling. This is not expected behavior. The child should be referred to a competent professional to deal with his aggression so that an accurate assessment can be made and a care plan determined. Outward aggression to others is not normal behavior and should be evaluated. The validation will be helpful for the referral, but the referral is the priority action. This may be recommended by the professional once an accurate assessment is made.

Kimberly, age 4 years, sometimes wakes her parents up at night, screaming, thrashing, sweating, and apparently frightened. Yet she is not aware of her parents' presence when they check on her. She lies down and sleeps without any parental intervention. This most likely scenario is: A: nightmares. B: sleep terrors. C: seizure activity. D: sleep apnea.

B: sleep terrors. In sleep terrors, the child is only partially aroused; therefore, she does not remember her parents' presence. A nightmare is a frightening dream followed by full awakening; therefore, the child would realize that her parents are present. The description of the child's experience does not indicate the presence of seizure activity. Sleep apnea is a cessation of breathing during sleep.

Infants most at risk for sudden infant death syndrome (SIDS) are those (Select all that apply.) A) Who sleep supine B) Who sleep prone C) Who were preterm D) With prenatal drug exposure E) With a cousin that died of SIDS

BCD

The nurse is assessing a newborn's reflexes. Stroking the outer sole of the foot assesses which reflex? Perez Babinski Glabellar Dance or step

Babinski This is a description of the Babinski reflex. This reflex disappears after 1 year; if present beyond 1 year of age, it may indicate neurologic deficit(s).

A child in renal failure has hyperkalemia. Which foods should be avoided?

Bananas, carrots, and green leafy vegetables.

3-year-old Fine Motor Development

Builds tower of 9-10 cubes Builds bridge with three cubes Adeptly places small pellets in narrow-necked bottle In drawing, copies circle, imitates cross, names what has been drawn; cannot draw stick figure but may make circle with facial features

The nurse is concerned with the prevention of communicable disease. Primary prevention results from A) early diagnosis. B) strict isolation. C) immunizations. D) treatment of disease.

C

The parents of a 4½-year-old girl are worried because she has an imaginary playmate. Based on the nurse's knowledge of the preschooler, the most appropriate response is: A: a psychosocial evaluation is indicated. B: an evaluation of possible parent-child conflict is indicated. C: having imaginary playmates is normal and useful at this age. D: having imaginary playmates is abnormal after age 2 years.

C: having imaginary playmates is normal and useful at this age. Imaginary playmates are a part of normal development at this age and serve many purposes, including being a friend in times of loneliness, accomplishing what the preschooler is still attempting, and experiencing what the preschooler wants to forget or remember. Because an imaginary playmate is part of normal development, a psychosocial evaluation is not warranted. Because an imaginary playmate is part of normal development, an evaluation of the parent-child relationship is not warranted. Imaginary playmates are commonly present during the preschool years; therefore, they are not abnormal after the age of 2 years.

According to Erikson, the primary psychosocial task of the preschool period is developing a sense of: A: identity. B: intimacy. C: initiative. D: industry.

C: initiative. Preschoolers focus on developing initiative. The stage is known as initiative versus guilt. Identity versus role confusion is associated with adolescence. Intimacy versus isolation is associated with young adulthood. Industry versus inferiority is associated with the school-aged child.

During a well-child visit, the father of a 4-year-old child tells the nurse that he is not certain if his child is ready for kindergarten. The child's birthday is close to the cutoff date, and the child has not attended preschool. The most appropriate recommendation by the nurse is to: A: encourage the father to have the child start kindergarten. B: recommend to the father that he postpone kindergarten and send the child to preschool. C: refer the child for developmental screening and make a recommendation based on the results. D: have the father observe a kindergarten class and then decide if his child would enjoy the experience.

C: refer the child for developmental screening and make a recommendation based on the results. A developmental screening will provide the necessary information to help the family determine readiness. Encouraging the father to have the child start kindergarten does not address the father's concern about readiness and suggests that his concerns are not warranted. Recommending to the father that he postpone kindergarten and send the child to preschool assumes that the child is not ready for kindergarten, but the recommendation is not based on any data or facts. Recommending to the father that he simply place his child in preschool may lead to the child's boredom with school. Having the father observe a kindergarten class and then decide if the child would enjoy the experience will provide information about kindergarten but not about whether his child is ready to begin and thrive there.

What is the priority nursing intervention for a 3-year-old in a hip spica cast?

Check circulation, sensation, and motion of toes.

Spiritual Development

Children at this age think in concrete terms but are avid learners and have a great desire to learn about their God or deity. They picture God as human and use adjectives such as "loving" and "helping" to describe their deity. They are fascinated by the concepts of hell and heaven, with a developing conscience and concern about rules, and they may fear going to hell for misbehavior. School-age children want and expect to be punished for misbehavior and, when given the option, tend to choose a punishment that "fits the crime." Often they view illness or injury as a punishment for a real or imagined misdeed. The beliefs and ideals of family and religious persons are more influential than those of their peers in matters of faith.

What option should be stressed in a health promotion plan for new parents?

Children should not be left alone in height chairs even if the chair is properly locked with the tray table secured.

What is the most common cause of secondary hyperparathyroidism?

Chronic renal disease.

The nurse is using a bulb syringe to suction a neonate after delivery. Which of the following is an important consideration? Compress bulb after insertion. Clear pharynx before nasal passages. Use two bulb syringes, one for pharynx and one for nares. Use bulb syringe until secretions are removed because mechanical suction is contraindicated.

Clearing the pharynx before the nasal passages will minimize the potential aspiration of amniotic fluid.

Hodgkin Lymphoma

Clinical manifestations: ○ Painless enlargement of lymph nodes ○ Most common finding is enlarged, firm, nontender, movable nodes in the supraclavicular or cervical area

What should be included in the postoperative care of a preschool child who had a brain tumor removed?

Close supervision while regaining consciousness.

What can thyroid storm progress to if medical interventions are not successful?

Coma.

A 3-month-old bottle-fed infant is allergic to cow's milk. Which is the best substitute to teach the parents to use? A) Goat's milk B) Soy-based formula C) Skim milk diluted with water D) Casein hydrolysate milk formula

D

A 9-month-old infant is seen in the emergency department after developing urticaric rash with cough and wheezing. When collecting the history of events prior to the sudden onset of the rash with cough and wheezing, the mother states they were "feeding the baby new foods." Which food is the possible cause of this type of reaction in the infant? A) Potatoes B) Green beans C) Spinach D) Peanut butter

D

By what age would the nurse expect most children to use sentences of four or five words? A: 18 months B: 24 months C: 3 years D: 4½ years

D: 4½ years Children ages 4 to 5 years use sentences of four or five words. An 18-month-old child has a vocabulary of approximately 10 words. A 24-month-old child uses two- or three-word phrases. A 3-year-old child uses sentences of three or four complete words.

Ashley, age 4½ years, is afraid of dogs. What should the nurse recommend to her parents to help her with this fear? A: Keep her away from dogs. B: Buy her a stuffed dog toy. C: Force her to touch a dog briefly. D: Let her watch other children play with a dog.

D: Let her watch other children play with a dog. The parents should actively seek ways to deal with fear. By observing other children at play with dogs, the child can adapt. Keeping their child away from dogs avoids the object of fear rather than addressing the fear and finding solutions. Buying a child a stuffed dog toy avoids the object of fear rather than addressing the fear and finding solutions. Forcing the child to touch a dog without working up to it may increase the level of fear.

What is a secondary effect when a child experiences decreased muscle strength, tone, and endurance from immobilization?

Decreased exercise tolerance.

What is the most important nursing consideration related to congenital hypothyroidism?

Early identification of the disorder.

What is the rationale for elevating an extremity after a soft tissue injury such as a sprained ankle?

Elevation reduces edema formation.

Implementing Discipline

Emphasizes consistency in disciplinary actions.

The nurse is planning care for a child recently diagnosed with diabetes insipidus. Which nursing intervention should be planned?

Encourage the child to wear medical identification.

What is an appropriate nursing intervention for a 2-year-old child who is unconscious but stable after a car accident?

Encourage the parents to hold, talk to, and sing to the child as they usually would

A mother planned to breastfeed her infant before giving birth at 35 weeks of gestation. The infant is stable and receiving oxygen. Which of the following is the most appropriate nursing action related to feeding this infant? Assist mother in expressing breast milk. Evaluate infant's ability to breastfeed. Explain to mother that infant is too small to receive breast milk. Reassure mother that infant formula is a good alternative to breastfeeding.

Evaluate infant's ability to breastfeed. Preterm infants are able to breastfeed when they demonstrate readiness. A careful evaluation of readiness includes behavioral state, presence of efficient suck reflex, maintenance of body temperature, and respiratory status.

An infant is born with ambiguous genitalia. Tests are being done to assist in gender assignment. The parents tell the nurse that family and friends are asking what caused the baby to be this way. What should the nurse's explanation include?

Explain the disorder so that the parents can explain it to others.

Communicable Diseases

Focuses on preventing the spread of diseases. ○ Primary prevention is immunizations ○ Most important preventative measure is hand washing

A neonate with a goiter has just been admitted to the newborn nursery. A priority nursing intervention is to

Have a tracheostomy set at bedside.

Superficial palpation of the abdomen is often perceived by the child as tickling. Which measure by the nurse is most likely to minimize this sensation and promote relaxation? Palpate another area simultaneously. Ask the child not to laugh or move if it tickles. Begin with deeper palpation and gradually progress to superficial palpation. Have the child "help" with palpation by placing his or her hand over the palpating hand.

Have the child "help" with palpation by placing his or her hand over the palpating hand. Having the child "help" allows the nurse to perform the assessment while including the child in his or her care. Palpating another area simultaneously would not promote relaxation and would make it more difficult to perform the abdominal assessment. Asking a child not to laugh or move if it tickles may only contribute to the child's laughter or may prove frustrating to both the child and the nurse. Deeper palpation will enhance the "tickling" sensation, not lessen it.

What should be the first action when a child has urinary incontinence?

Have the child evaluated for possible urinary tract infection.

Disorders of sexual development (DSD) present unique challenges to both the patient, family members and health care providers. In preparing for health promotion materials to be used in a teaching session for health care providers on this subject, which information should be included?

Health care providers should understand their own feelings and implicit bias relative to DSD.

What laboratory finding indicates nephrosis?

Hypoalbuminemia.

3. Based on Piaget's theory of cognitive development, what is one basic concept a child is expected to attain during the first year of life? • If an object is hidden, that does not mean that it is gone. • He or she cannot be fooled by changing shapes. • Parents are not perfect. • Most procedures can be reversed.

If an object is hidden, that does not mean that it is gone.

Which of the following factors makes the neonate more prone to problems of dehydration and acidosis? Immature lungs alter neonate's ability to regulate fluid balance. Immature kidneys cannot concentrate urine. Rate of fluid exchange is less than in adults. Rate of metabolism is less in relation to body weight.

Immature kidneys cannot concentrate urine. The kidneys' immature state interferes with the regulatory effect that the kidneys perform in adults, thus increasing the risk of dehydration and acidosis.

What is the treatment for diabetic ketoacidosis (DKA)?

Immediate treatment is required as it is a life-threatening situation.

What condition may be suspected in an adolescent male with elevated growth hormone levels?

Pituitary adenoma.

The nurse notices that a toddler is more cooperative taking medicine from a small cup than from a large cup. This is an example of which characteristic of preoperational thought? Egocentrism Irreversibility Inability to conserve Transductive reasoning

Inability to conserve The smaller cup makes it look like less medicine to the child at this stage of cognitive development. The inability to see situations from other perspectives, besides their own, does not facilitate medication administration. The inability to reverse or undo actions physically initiated does not facilitate medication administration. Focusing on particulars does not explain the cooperation with the smaller medication cup.

Medication Administration

Includes checking dosage and identification of patient

How does increased exercise affect the management of type 1 diabetes mellitus?

Increased food intake is expected.

Parents of a newborn bring their male son to the emergency room. The infant appears fretful and the parents state that he has not voided in several hours. Inspection of the penis reveals edema and the nurse is unable to retract the foreskin. Based on this assessment, what would the nurse anticipate as the priority action?

Inform the ER physician of the patient's condition.

Which of the following interventions may decrease the incidence of physiologic jaundice in a healthy term infant? Institute early and frequent feedings. Place infant's crib near window for exposure to sunlight. Bathe infant when axillary temperature is 36.3�C (97.3�F). Suggest that mother initiate breastfeeding when danger of jaundice is past.

Institute early and frequent feedings. Early, frequent feedings in full-term infants increases intestinal motility, enhancing the excretion of unconjugated bilirubin.

What does the presence of Doll's reflex indicate in the assessment of neurological integrity of the eyes?

Intact cranial nerve III.

Which statement best describes Cushing syndrome?

It is caused by excessive production of cortisol.

What nursing intervention should be included in the care of a school-age child with bacterial meningitis?

Keep environmental stimuli to a minimum.

Which of the following findings on physical assessment of a neonate would indicate the need for further observation and examination? Epstein pearls Cyanotic hands and feet Babinski reflex Low-set ears

Low-set ears

Respiratory Syncytial Virus

Maintain adequate fluid intake, airway maintenance, and medications. Hospitalization is recommended for children with respiratory distress or those with poor feeding, lethargy, and dehydration.

Which of the following is important in providing a neutral thermal environment for a low-birth-weight infant in an incubator? Use wool blankets. Avoid using disposable diapers. Maintain high humidity atmosphere. Closely monitor both incubator and rectal temperatures.

Maintain high humidity atmosphere.

Cystic Fibrosis

Management of gastrointestinal problems: -Replacement of pancreatic enzymes, which are administered with meals and snacks to ensure that digestive enzymes are mixed with food in the duodenum. Enteric-coated products prevent the neutralization of enzymes by gastric acids, thus allowing activation to occur in the alkaline environment of the small bowel. -Children with CF require a well-balanced, high-protein, high-caloric diet (because of their impaired intestinal absorption). In fact, a group of experts recommend that children and adolescents with CF who are 2 to 20 years of age should have an energy intake of 110% to 200% of standards for healthy persons Vitamin supplementation: -Vitamins A, D, E, and K need to be supplemented in higher doses.

What does the callus that develops at a fracture site provide?

Means for holding bone fragments together.

What is an appropriate nursing intervention for a child with chronic osteomyelitis?

Move and turn the child carefully and gently to minimize pain.

What is an important nursing consideration when caring for a child with end-stage renal disease?

Multiple stresses are placed on children with ESRD and their families because their lives are maintained by drugs and artificial means.

Complications With The Immobilized Child

Muscular System: -Decreased muscle strength, tone, and endurance -Disuse atrophy and loss of muscle mass -Loss of joint mobility -Weak back muscles -Weak abdominal muscles Skeletal System: -Bone demineralization, osteoporosis, and hypercalcemia. -Negative bone calcium uptake. Metabolism: -Decreased metabolic rate -Negative nitrogen balance -Hypercalcemia -Decreased production of stress hormones Cardiovascular System: -Decreased efficiency of orthostatic neurovascular reflexes -Diminished vasopressor mechanism -Altered distribution of blood volume -Venous stasis -Dependent edema Respiratory System: -Decreased need for oxygen -Decreased chest expansion and diminished vital capacity -Poor abdominal tone and distention -Mechanical or biochemical secretion retention -Loss of respiratory muscle strength Urinary System: -Alteration of gravitational force -Impaired ureteral peristalsis Integumentary System: -Altered tissue integrity

Why are infants particularly vulnerable to acceleration-deceleration head injuries?

Musculoskeletal support of head is insufficient

Why are infants particularly vulnerable to acceleration-deceleration head injuries?

Musculoskeletal support of the head is insufficient.

What is one of the etiologies associated with central precocious puberty?

Neoplasm.

What do the presence of Moro, tonic neck, and withdrawal reflexes suggest in a 2-month-old infant after a car accident?

Neurologic health

A 6-year-old is hospitalized with a fractured femur. Based on the nurse's knowledge of pain assessment tools and child development, which assessment tools are most appropriate for this age child? Select all that apply. Oucher scale CRIES scale Poker chip tool Faces pain scale Postoperative pain score

Oucher scale Poker chip tool Faces pain scale A 6-year-old should be able to use the Oucher scale to choose which face best matches their pain level. A 6-year-old should be able to count and understand the concept of the Poker chip tool. A 6-year-old should be able to use the Faces pain scale to choose which face best matches their pain level. The CRIES scale was developed for assessing pain in the preterm and term neonate. The Postoperative pain score assesses pain in 1- to 7-month-old-infants.

The nurse is using the C.R.I.E.S. pain assessment tool on a preterm infant in the neonatal intensive care unit (NICU). Which is a component of this tool? Color Reflexes Oxygen saturation Posture of the arms and legs

Oxygen saturation The components of the scale are Crying, Requires increased oxygen. Increased vital signs, Expression, and Sleepless. Changes in oxygen saturation would affect scoring. Color is not a component of this scale. Reflexes are not a component of this scale. Posture of the arms and legs is not a component of this scale.

Recurrent Pain

Pain that is episodic and recurs is defined as recurrent pain—the time frame within which episodes of pain recur every 3 months or more frequently. Recurrent pain syndromes in children include migraine headache, episodic sickle cell pain, recurrent abdominal pain (RAP), and recurrent limb pain.

A nurse working in triage in the emergency room is assessing a pediatric patient, age 4, who presents with pain in the wrist. The patient refuses to move the involved extremity. Questioning of the patient and parent reveal no trauma event. Based on this observation the nurse suspects that the patient may have

Partial dislocation or subluxation.

A nurse is caring postoperatively for an 8-year-old child with multiple fractures and other trauma resulting from a motor vehicle injury. The child is experiencing severe pain. Which is an important consideration in managing the child's pain? Give only an opioid analgesic at this time. Increase the dosage of analgesic until the child is adequately sedated. Plan a preventive schedule of pain medication around the clock. Give the child a clock and explain when he or she can have pain medications.

Plan a preventive schedule of pain medication around the clock. An around-the-clock administration strategy should be used for a child recovering from trauma and surgery. This schedule will help prevent low plasma levels of the drug, leading to breakthrough pain. It is appropriate for the immediate concern of the child's pain to give an opioid analgesic, but this will not facilitate the more long-term plan of pain management. The dosage of analgesic is increased until the pain is controlled, not until sedation is adequate. The child should be frequently assessed for pain and medication doses titrated accordingly. It is inappropriate to give a child a clock with instructions as to when pain medication can be given, especially a child who has experienced a traumatic event.

What are major goals of therapeutic management for juvenile idiopathic arthritis (JIA)?

Prevent physical deformity; preserve joint function.

Infective Endocarditis

Primary intervention: ○ Treatment should be instituted immediately and consists of administration of high doses of appropriate antibiotics IV for 2 to 8 weeks ○ Frequent echocardiograms are done to monitor for vegetations, valve functions, and ventricular function ○ Heart surgery may be necessary to repair or replace the valve

Osteosarcoma

Primary tumor site: Lower extremities (femur)

Nursing care for the neonate with caput succedaneum should include which of the following? Monitor for signs of shock. Monitor for signs of infection. Reassure family that no specific treatment is needed. Reassure family that swelling will resolve within 3 months.

Reassure family that no specific treatment is needed. Caput succedaneum, an area of swelling above the bones of the skull, can occur in a vertex delivery. It usually subsides within a few days with no intervention.

The nurse is caring for a child hospitalized with acute adrenocortical insufficiency. The acute phase seems to be over when ascending flaccid paralysis occurs. What is the most appropriate nursing action?

Reassure the family that this condition is temporary.

Which statement explains why it can be difficult to assess a child's dietary intake? No systematic assessment tool has been developed for this purpose. Biochemical analysis for assessing nutrition is expensive. Families usually do not understand much about nutrition. Recall of children's food consumption is frequently unreliable.

Recall of children's food consumption is frequently unreliable. It is difficult for parents to recall exactly what their child has eaten. Concurrent food diaries are somewhat more reliable. Systematic tools have been developed and are available. Nutrients for different foods are known; the quantity and type of food consumed are the facts that are difficult to ascertain. The family does not need nutritional knowledge to describe what the child has eaten.

While nipple feeding a high-risk neonate, the nurse observes occasional apnea, pallor, and bradycardia. This has not occurred with previous feedings. The most appropriate nursing action is which of the following? Resume gavage feeding until asymptomatic. Let neonate rest before nipple feeding again. Recognize that this may indicate an underlying illness. Use a high-flow, pliable nipple because it requires less energy to use.

Recognize that this may indicate an underlying illness. Poor feeding behaviors in a neonate who has previously fed without difficulty may indicate an underlying problem and should be further investigated.

Family-centered care

Recognizes the family as the constant in a child's life and supports visitation and trust-building with families.

What is vesicoureteral reflux usually associated with?

Recurrent kidney infections.

The nurse is caring for a 12-year-old child who sustained major burns when putting charcoal lighter on a campfire. The nurse observes that the child is "very brave" and appears to accept pain with little or no response. What is the most appropriate nursing action? Request a psychological consultation. Ask why the child does not have pain. Praise the child for the ability to withstand pain. Encourage continued bravery as a coping strategy.

Request a psychological consultation. A psychological consultation will assist the child in verbalizing fears. This age-group is very concerned with physical appearance. The psychologist can help integrate the issues the child is facing. It is likely that the child is having pain but not acknowledging the pain. Speaking with a psychologist might assist the child in relaying fears and pain. If the child is feeling pain, the nurse should not offer praise for hiding the pain. The nurse should encourage the child to speak up during painful episodes so that the pain can be managed appropriately. Bravery may not be an effective coping strategy if the child is in severe pain.

18 Months Gross Motor Development

Runs clumsily; falls often Walks up stairs with one hand held Pulls and pushes toys Jumps in place with both feet Seats self on chair Throws ball overhand without falling

An expectation of the patient in a health care setting in terms of charting and documentation is that? Information will be shared only with physicians in the hospital or clinic setting regardless of whether they are taking care of the patient. The use of nursing informatics requires that passwords be changed upon access to maintain patient confidentiality. The patient is assured that anyone in the hospital facility can access their chart. Safeguard systems are in place within the hospital or clinic setting to help maintain confidentiality of patient records.

Safeguard systems are in place within the hospital or clinic setting to help maintain confidentiality of patient records. Safeguard systems are in place, regardless of the practice setting so as to assure that health care providers who access records do so to maintain confidentiality of patient records. Only health care providers that are involved in the patient's care should be accessing their medical records. Nursing informatics is the application of computer usage with regard to health care information. Although password changes are a part of the safeguard integrity of systems, they are not typically changed with each access attempt.

The parents of a toddler ask the nurse for suggestions about discipline. When discussing the use of timeouts, which of the following suggestions should the nurse include? Send the child to his or her room. If the child cries, refuses, or is more disruptive, try another approach. Select an area that is safe and nonstimulating, such as a hallway. The general rule for length of time is 1 hour per year of age.

Select an area that is safe and nonstimulating, such as a hallway. The area must be nonstimulating and safe. The child becomes bored in this environment and then changes his or her behavior to rejoin activities. The child's room may have toys and other forms of amusement that may negate the effect of being separated from family activities. When the child engages in this type of behavior, the timeout begins when the child quiets. The general rule is 1 minute per year. An hour per year is excessive.

13. According to Piaget, at what stage of development do children typically solve problems through trial and error? • Sensorimotor stage • Preoperational stage • Formal operational stage • Concrete operational stage

Sensorimotor stage

Chlamydia Infection

Symptoms: Male: Meatal erythema, tenderness, itching, dysuria, urethral discharge, or no symptoms Female: Mucopurulent cervical exudate with erythema, edema, congestion, or no symptoms Treatment: -Single oral dose of azithromycin or 7 days of oral doxycycline administered twice daily -*If pregnant, azithromycin Education: -Instruct patient to abstain from sexual intercourse for 7 days after single-dose treatment. -Test and treat for other STIs. -Find and treat sexual contacts. -Educate young people on facts of the disease and its spread. -Encourage use of condoms in sexually active young people. -Educate that repeat infection increases risk for PID. -Rescreen pregnant women 3 weeks after treatment.

What is helpful during this period of immobilization for a preschool child?

Take the child for a "walk" by wagon outside the room.

Injury Prevention: Drowning

Teach child to swim. Teach basic rules of water safety. Select safe and supervised places to swim. Check sufficient water depth for diving. Caution child to swim with a companion. Ensure that child uses an approved flotation device in water or boat. Advocate for legislation requiring fencing around pools. Learn cardiopulmonary resuscitation.

What should the nurse do if a 17-year-old with type 1 diabetes mellitus starts drinking alcohol?

Teach the adolescent about the effects of alcohol on type 1 diabetes mellitus and how to prevent problems associated with alcohol intake.

How should the nurse respond to parents expressing guilt over a submersion injury?

Tell me more about your feelings.

2. A nurse is knowledgeable about both growth and development. Which assessment finding indicates the child's development is on target? • The child has not gained weight for 3 months. • The child can throw a large ball but not a small ball. • The child's arms are the most rapidly growing part of the child's body. • The child can pull herself or himself to her or his feet before the child is able to sit steadily.

The child can throw a large ball but not a small ball.

In teaching a group of nursing students about factors that could lead to the development of urinary tract infections, which critical aspect should the nursing instructor focus on?

The concept of urinary stasis.

Basic treatment of musculoskeletal soft tissue injuries involves the use of RICE modalities during the first 12 to 24 hours. Which option would be included in RICE?

The extremity should be maintained in proper alignment and activity should be limited.

A child is receiving cyclosporine following a kidney transplant. The child's parents ask the nurse the reason for the cyclosporine. What is the nurse's response based on?

The medication's purpose is to suppress rejection.

APGAR Scoring

The most frequently used method to assess newborns' immediate adjustment to extrauterine life is the Apgar scoring system which is tested at 1 minute, 5 minute, and repeated until newborn is stabilized. Total scores: 0 to 3: Severe distress 4 to 6: Moderate difficulty 7 to 10: Absence of difficulty in adjusting to extrauterine life. Appearance: 0: Pale/blue all over 1: Acrocyanosis (Pink body, blue hands/feet) 2: Pink all over Pulse: 0: Absent 1: <100 bpm 2: >100 bpm Grimace: 0: No response to stimulation 1: Grimace (no cry) to stimulation 2: Cry & active movement to stimulation Activity: 0: None, flaccid 1: Some flexion of arms & legs 2: Arms & legs flexed Respiratory: 0: Absent 1: Weak, irregular cry 2: Strong, vigorous cry

The nurse is teaching the parent of a 2-year-old child how to care for the child's teeth. Which instruction should be included? Flossing is not recommended at this age. Toddlers are old enough to brush their teeth effectively. The parent should brush the toddler's teeth with plain water if he or she does not like toothpaste. The toddler's toothbrush should be small and have hard, rounded, nylon bristles.

The parent should brush the toddler's teeth with plain water if he or she does not like toothpaste. Some toddlers do not like the flavor of toothpaste, so water can be used for teeth brushing at this age. Flossing should be done after brushing to establish it as part of dental care for the toddler. Two-year-olds cannot effectively brush their own teeth; parental assistance is necessary. Soft multitufted bristled toothbrushes are recommended to avoid damaging a toddler's teeth or gums.

Urine specimen results for a pediatric patient note greater than 100,000 colony forming units (CFUs) but the patient denies any complaints with urination. What would the nurse suspect based on this information?

The patient has asymptomatic bacteriuria.

Which statement is most characteristic of the motor skills of a 24-month-old child? The toddler walks alone but falls easily. The toddler's activities begin to produce purposeful results. The toddler is able to grasp small objects but cannot release them at will. The toddler's motor skills are fully developed but occur in isolation from the environment.

The toddler's activities begin to produce purposeful results. Gross and fine motor mastery occurs with other activities that have a purpose, such as walking to a particular location or putting down one toy and picking up a new toy. By 2 years of age, children are able to walk up and down stairs without falling. Grasping small objects without being able to release them is a task of infancy. Interaction with the environment is essential for mastery of both fine and gross motor skills at this age and beyond.

Systemic Lupus Erythematosus (SLE)

Therapeutic Management: -The goal of treatment is to ensure the child's health by balancing the medications necessary to avoid exacerbation and complications while preventing or minimizing treatment-associated morbidity. -Therapy involves the use of specific medications and general supportive care. -The drugs used to control inflammation are: Corticosteroids administered in doses sufficient to control inflammation and then tapered to the lowest suppressive dose or given intravenously during acute flares. Hydroxychloroquine, an antimalarial, is a useful medication for inflammatory control, rash, and arthritis. NSAIDs relieve muscle and joint inflammation, and immunosuppressive agents, such as cyclophosphamide, are administered for renal and CNS disease. Mycophenolate, azathioprine, and methotrexate are effective immunosuppressive drugs that may be used to control SLE and allow steroids to be reduced. Rituximab is a monoclonal antibody that results in decreased antibody formation and has been used off-label in pediatric patients with lupus who have not responded to standard therapy. Antihypertensives, low-dose aspirin (as a blood thinner), and calcium and vitamin D supplements are just a few of the additional remedies that may be necessary to treat or avoid complications. -General supportive care includes sufficient nutrition, sleep and rest, and exercise. Exposure to the sun and ultraviolet B (UVB) light is limited because of its association with SLE exacerbation.

Which physiologic alteration is characterized by destruction of pancreatic beta cells that produce insulin?

Type 1 diabetes.

What possible complications should be assessed in a toddler with acute renal failure?

Water intoxication.

What is the best way to detect fluid retention in a non-potty-trained child with nephrotic syndrome?

Weigh the child daily.

When is the optimum time for an infant with a lower limb deficiency to be fitted with a prosthetic device?

When the infant is developmentally ready to stand up.

Can a child with type 1 diabetes mellitus participate in sports?

Yes, exercise is not restricted unless indicated by other health conditions.

Transdermal fentanyl (Duragesic) is being used for an adolescent with cancer who is in hospice care. The adolescent has been comfortable for several hours but now complains of severe pain. The most appropriate nursing action is to administer meperidine (Demerol) intramuscularly. administer morphine sulfate immediate release (MSIR) intravenously. use a nonpharmacologic strategy. place another fentanyl (Duragesic) patch on the adolescent.

administer morphine sulfate immediate release (MSIR) intravenously. The nurse should administer an immediate-release opioid intravenously, such as MSIR, for the breakthrough pain. Intramuscular (IM) injections should be avoided in cancer patients because of an increased risk of bleeding and the fact that IM injections do not act immediately. Nonpharmacologic strategies will not be effective in severe pain. Transdermal fentanyl will take up to 24 hours to reach peak effect and therefore is not effective for severe breakthrough pain.

4. A nurse is examining a toddler and is discussing with the mother psychosocial development according to Erikson's theories. Based on the nurse's knowledge of Erikson, the most age-appropriate activity to suggest to the mother at this stage is to • feed lunch. • allow the toddler to start making choices about what to wear. • allow the toddler to pull a talking-duck toy. • turn on a TV show with bright colors and loud songs.

allow the toddler to start making choices about what to wear.

One of the major tasks of toddlerhood is toilet training. In teaching the parents about a child's readiness for toilet training, it is important for the nurse to emphasize that nighttime bladder control develops first, so parents should focus on that in the initial teaching with their toddler. bowel control is accomplished before bladder control, so the parent should focus on bowel training first. the toddler must have the gross motor skill to climb up to the adult toilet before training is begun. the universal age for toilet training to begin is 2 years, and the universal age for completion is 4 years.

bowel control is accomplished before bladder control, so the parent should focus on bowel training first. Bowel training is usually accomplished before bladder training because of its greater regularity and predictability. The sensation to defecate is stronger than that of urination. The completion of bowel training will give the toddler a sense of accomplishment that can be carried onto bladder training. Nighttime bladder control normally takes several months to years after daytime training; therefore, this should not be the initial focus of toilet training with a toddler. There is no universal right age to begin toilet training or an absolute deadline to complete training. One of the nurse's most important responsibilities is to help parents identify the readiness signs in their child.

The nurse is assessing skin turgor in a child. The nurse grasps the skin on the abdomen between the thumb and index finger, pulls it taut, and quickly releases it. The tissue remains suspended, or tented, for a few seconds, and then slowly falls back on the abdomen. Based on the nurse's knowledge of assessing skin turgor, the assessment finding is that the tissue shows normal elasticity. child is properly hydrated. assessment is done incorrectly. child has poor skin turgor.

child has poor skin turgor. Tenting is the term for poor skin turgor. In normal elasticity, the skin would return immediately to its original position. If the child is properly hydrated, skin turgor would be elastic. The correct way to assess turgor is to grasp the skin on the abdomen between the thumb and index finger, pull it taut, and quickly release it.

Which of the following findings would the nurse consider normal in assessing the anterior fontanel of a neonate? Flat Closed Sunken Bulges when infant is asleep

flat

Which urine test would be considered abnormal?

pH:4

A child is being seen in the emergency department with multiple facial abrasions and lacerations. The combination agent lidocaine, adrenaline, and tetracaine (LAT) is applied topically to the wounds. The purpose of this combination therapy is to cleanse the wound. promote scab formation. prevent infection of the wound. provide anesthesia to the wound.

provide anesthesia to the wound. The combination of lidocaine, adrenaline, and tetracaine provides anesthesia within 10 to 15 minutes of application. LAT does not have a cleansing effect, effect on scab formation or antibacterial effect.

Characteristics of physical development of a 30-month-old child are Select all that apply. anterior fontanel is open. birth weight has doubled. genital fondling is noted. sphincter control is achieved. primary dentition is complete.

sphincter control is achieved. primary dentition is complete. Sphincter control in preparation for bowel and bladder control is usually achieved by 30 months of age. Primary dentition is usually completed by 30 months of age. Anterior fontanel closes between 12 and 18 months of age. Birth weight should double at 5-6 months of age and quadruple by 2½ years of age. Genital fondling is not a characteristic of physical development of this age group. This is part of the development of gender identity.

20. Which statements provides the best description of parallel play? (Select all that apply.) • Two children playing checkers together. • One child playing with his truck while another child plays with a car while seated on the floor. • Three children playing each playing with a deck of cards but performing different actions with the respective deck of cards. • Two children playing with dolls together while a third child walks by with a doll stroller and asks if she could play with them.

• Two children playing checkers together. • Three children playing each playing with a deck of cards but performing different actions with the respective deck of cards.

19. During a well-baby visit, the parents of a 12-month-old ask the nurse for advice on age-appropriate toys for their child. Based on the nurse's knowledge of developmental levels, the most appropriate toys to suggest are (Select all that apply.) • push-pull toys. • toys with black-white patterns. • pop-up toy such as Jack-in-the-box. • soft toys that can be put in the mouth. • toys that pop apart and go back together.

• push-pull toys. • pop-up toy such as Jack-in-the-box. • toys that pop apart and go back together.

Food Sensitivity: Food allergies

○ Adverse immunologic reactions to foods ○ Clinical manifestations: Systemic: Anaphylaxis, growth failure GI: Abdominal pain, vomiting, cramping, diarrhea Respiratory: Cough, wheezing, rhinitis, infiltrates Cutaneous: Urticaria, rash, atopic dermatitis

Nursing Process

● Assessment- recognize cues ● Diagnosis- analyze cues/prioritize hypotheses ● Planning- generate solutions ● Implementation- take action ● Evaluation- evaluate outcomes

Level of Consciousness

● Full consciousness- awake and alert, oriented to time, place and person ● Confusion- impaired decision making ● Disorientation- confusion regarding time, place, and or person; decreased LOC ● Lethargy- limited spontaneous movement, sluggish speech, drowsiness ● Obtundation- arousable with stimulation ● Stupor- remaining in a deep sleep, responsive only to vigorous and repeated stimulation ● Coma- no motor or verbal response to painful stimuli ● Persistent vegetative state- permanently lost function of the cerebral cortex

Child Abuse: Physical & Sexual

●Physical Abuse: -Physical abuse is the deliberate infliction of physical injury on a child and can include anything from bruises and fractures to brain damage. -Abusive head trauma (AHT) is a serious form of physical abuse caused by violent shaking of infants and young children. Other commonly used terms include shaken baby syndrome, inflicted head injury, or neuro-inflicted brain injury. This violent shaking is most often a result of the caregiver's frustration with crying, maternal stress, or depression. -Munchausen syndrome by proxy (MSBP) is a form of child abuse in which caregivers deliberately exaggerate or fabricate histories and symptoms or induce symptoms. It may include physical, emotional, and psychologic abuse for the gratification of the caregiver. -Three risk factors are commonly identified in child abuse: parental characteristics, characteristics of the child, and environmental characteristics. However, no single factor or group of factors is predictive of abuse. ●Sexual Abuse: Types: Incest: Any physical sexual activity between family members; blood relationship is not required Molestation: A vague term that includes "indecent liberties," such as touching, fondling, kissing, single or mutual masturbation, or oral-genital contact Exhibitionism: Indecent exposure, usually exposure of the genitalia by an adult man to children or women Child pornography: Arranging and photographing, in any media, sexual acts involving children, alone or with adults or animals, regardless of consent by the child's legal guardian; also may denote distribution of such material in any form. Child prostitution: Involving children in sex acts for profit and usually with changing partners Pedophilia: Refers to the preference of an adult for prepubertal children as the means of achieving sexual excitement.


Ensembles d'études connexes

Effective Leadership and Management - Midterm Combined

View Set

4 PASKAITA MOKSLINĖS LITERATŪROS APŽVALGA (I)

View Set